Surgical Infections Flashcards

1
Q

Transferrin plays a role in host defense by

A. Sequestering iron, which is necessary for microbial growth

B. Increasing the ability of fibrinogen to trap microbes

C. Direct injury to the bacterial cell membrane

D. Direct injury to the bacterial mitochondria

A

Answer: A

Once microbes enter a sterile body compartment (eg, pleural or peritoneal cavity) or tissue, additional host defenses
act to limit and/or eliminate these pathogens.

Initially, several primitive and relatively nonspecific host defenses act to
contain the nidus of infection, which may include microbes as
well as debris, devitalized tissue, and foreign bodies, depending on the nature of the injury.

These defenses include the physical barrier of the tissue itself, as well as the capacity of proteins, such as lactoferrin and transferrin, to sequester the critical microbial growth factor iron, thereby limiting microbial growth.

In addition, fibrinogen within the inflammatory
fluid has the ability to trap large numbers of microbes during the process in which it polymerizes into fibrin.

Within the peritoneal cavity, unique host defenses exist, including a diaphragmatic pumping mechanism whereby particles including microbes within peritoneal fluid arc expunged from the
abdominal cavity via specialized structures on the undersurface of the diaphragm.

Concurrently, containment by the
omentum, the so-called “gatekeeper” of the abdomen and intestinal ileus, serves to wall off infection.

However, the latter
processes and fibrin trapping have a high likelihood of contributing to the formation of an intra-abdominal abscess.

How well did you know this?
1
Not at all
2
3
4
5
Perfectly
2
Q
Which is NOT a component of systemic inflammatory
response syndrome (SIRS)?

A. Temperature

B. White blood cell (WBC) count

C. Blood pressure

D. Heart rate

A

Answer: C

Infection is defined by the presence of microorganisms in host tissue or the bloodstream.

At the site of infection the classic findings of rubor, calor, and dolor in areas such as the skin or subcutaneous tissue are common.

Most infections in normal individuals with intact host defenses are associated with these local manifestations, plus systemic manifestations such as elevated temperature, elevated white blood cell (WBC) count, tachycardia, or tachypnea.

The systemic manifestations noted above comprise the systemic inflammatory response syndrome (SIRS).

How well did you know this?
1
Not at all
2
3
4
5
Perfectly
3
Q

The best method for hair removal from an operative field is

A. Shaving the night before

B. Depilating the night before surgery

C. Shaving in the operating room

D. Using hair clippers in the operating room

A

Answer: D

Hair removal should take place using a clipper rather than a razor; the latter promotes overgrowth of skin microbes in small nicks and cuts.

Dedicated use of these modalities clearly
has been shown to diminish the quantity of skin microflora.
(See Schwartz 10th cd.,p. 141.)

How well did you know this?
1
Not at all
2
3
4
5
Perfectly
4
Q

A patient with necrotizing pancreatitis undergoes computed tomography (CT)-guided aspiration, which results in growth of Escherichia coli on culture. The most appropriate treatment is

A. Culture-appropriate antibiotic therapy

B. Endoscopic retrograde cholangiopancreatography with sphincterotomy

C. CT-guided placement of drain(s)

D. Exploratory laparotomy

A

Answer: D

The primary precept of surgical infectious disease therapy
consists of drainage of all purulent material, debridement of
all infected, devitalized tissue, and debris, and/or removal of foreign bodies at the site of infection, plus remediation of the
underlying cause of infection.

A discrete, walled-off purulent fluid collection (ie, an abscess) requires drainage via percutaneous drain insertion or an operative approach in which incision and drainage take place.

An ongoing source of contamination (eg, bowel perforation) or the presence of an
aggressive, rapidly spreading infection (eg, necrotizing soft
tissue infection) invariably requires expedient, aggressive operative intervention, both to remove contaminated material and infected tissue (eg, radical debridement or amputation) and to remove the initial cause of infection (eg, bowel resection).

How well did you know this?
1
Not at all
2
3
4
5
Perfectly
5
Q

Which factor does NOT influence the development of surgical site infections (SSIs)?

A. Duration of procedure

B. Degree of microbial contamination of the wound

C. Malnutrition

D. General anesthesia

A

Answer: D

Surgical site infections (SSIs) are infections of the tissues, organs, or spaces exposed by surgeons during performance of an invasive procedure.

SSIs are classified into incisional and
organ/space infections, and the former are further subclassified into superficial (limited to skin and subcutaneous tissue)
and deep incisional categories.

The development of SSIs is related to three factors: (1) the degree of microbial contamination of the wound during surgery,
(2) the duration of the
procedure, and
(3) host factors such as diabetes, malnutrition,
obesity, immune suppression, and a number of other underlying disease states.

How well did you know this?
1
Not at all
2
3
4
5
Perfectly
6
Q

During a laparoscopic appendectomy, a large bowel injury was caused during trochar placement with spillage of bowel contents into the abdomen. What class of surgical wound is this?

A. Class I (clean)

B. Class II (clean/contaminated)

C. Class III (contaminated)

D. Class IV (dirty)

A

Answer: C

Surgical wounds are classified based on the presumed magnitude of the bacterial load at the time of surgery.

Clean wounds (Class I) include those in which no infection is present; only skin microflora potentially contaminate the wound, and no
hollow viscus that contains microbes is entered. 

Class ID wounds are similar except that a prosthetic device (eg, mesh or valve) is inserted.

Clean/contaminatcd wounds (Class II) include those in which a hollow viscus, such as the respiratory, alimentary, or genitourinary tracts, with indigenous bacterial flora is opened under controlled circumstances without significant spillage of contents.

Contaminated wounds
(Class III) include open accidental wounds encountered early after injury, those with extensive introduction of bacteria into a normally sterile area of the body due to major breaks
in sterile technique (eg, open cardiac massage), gross spillage of viscus contents such as from the intestine, or incision
through inflamed, albeit nonpurulent, tissue.

Dirty wounds (Class IV) include traumatic wounds in which a significant delay in treatment has occurred and in which necrotic tissue is present, those created in the presence of overt infection as
evidenced by the presence of purulent material, and those
created to access a perforated viscus accompanied by a high
degree of contamination.

How well did you know this?
1
Not at all
2
3
4
5
Perfectly
7
Q

The most appropriate treatment of a 4-cm hepatic abscess is

A. Antibiotic therapy alone

B. Aspiration for culture and antibiotic therapy

C. Percutaneous drainage and antibiotic therapy

D. Operative exploration, open drainage of the abscess,
and antibiotic therapy

A

Answer: C

Hepatic abscesses are rare, currently accounting for approximately 15 per 100,000 hospital admissions in the United States.

Pyogenic abscesses account for approximately 80% of eases, the remaining 20% being equally divided among parasitic and fungal forms.

Formerly, pyogenic liver abscesses were caused
by pyelophlebitis due to neglected appendicitis or diverticulitis.

Today, manipulation of the biliary tract to treat a variety of diseases has become a more common cause, although in nearly
50% of patients no cause is identified.

The most common aerobic bacteria identified in recent series include E. coli, Klebsiella pneumoniae, and other enteric bacilli, enterococci, and
Pseudomonas spp., while the most common anaerobic bacteria are Bacteroides spp., anaerobic streptococci, and Fusobacterium spp.

Candida albicans and other similar yeasts cause the majority of fungal hepatic abscesses.

Snail (<1 cm), multiple
abscesses should be sampled and treated with a 4- to 6-week course of antibiotics.

Larger abscesses invariably are amenable to percutaneous drainage, with parameters for antibiotic therapy and drain removal similar to those mentioned above.

Splenic abscesses are extremely rare and are treated in a similar fashion.

Recurrent hepatic or splenic abscesses may require operative intervention—unroofing and marsupialization or
splenectomy, respectively.

How well did you know this?
1
Not at all
2
3
4
5
Perfectly
8
Q

Postoperative urinary tract infections (UTIs)

A. Are usually treated with a 7- to 10-day course of antibiotics.

B. Initial therapy should be directed by results of urine culture.

C. Are established by >104 CFU/mL of bacteria in urine culture in asymptomatic patients.

D. Can be reduced by irrigating indwelling Foley catheters daily.

A

Answer: B

The presence of a postoperative UTI should be considered based on urinalysis demonstrating WBCs or bacteria, a positive test for leukocyte esterase, or a combination of these elements. The diagnosis is established after >104 CFU/mL of microbes are identified by culture techniques in symptomatic patients, or > 10s CFU/mL in asymptomatic individuals.

Treatment for 3 to 5 days with a single antibiotic directed against
the most common organisms (eg, E. Coli, K. pneumoniae) that
achieves high levels in the urine is appropriate.

Initial therapy is directed by Gram’s stain results and is refined as culture results become available.

Postoperative surgical patients should have indwelling urinary catheters removed as quickly as possible, typically within 1 to 2 days, as long as they are
mobile, to avoid the development of a UTI.

How well did you know this?
1
Not at all
2
3
4
5
Perfectly
9
Q

The first step in the evaluation and treatment of a patient
with an infected bug bite on the leg with cellulitis, bullae, thin grayish fluid draining from the wound, and pain out
of proportion to the physical findings is

A. Obtain C-reactive protein

B. CT scan of the leg

C. Magnetic resonance imaging (MRI) of the leg

D. Operative exploration

A

Answer: D

The diagnosis of necrotizing infection is established solely
upon a constellation of clinical findings, not all of which are present in every patient.

Not surprisingly, patients often develop sepsis syndrome or septic shock without an obvious cause.

The extremities, perineum, trunk, and torso are most commonly affected, in that order.

Careful examination should be undertaken for an entry site such as a small break or sinus in the skin from which grayish, turbid semipurulent
material (‘dishwater pus”) can be expressed, as well as for the presence of skin changes (bronze hue or brawny induration),
blebs, or crepitus.

The patient often develops pain at the site
of infection that appears to be out of proportion to any of the physical manifestations.

Any of these findings mandates immediate surgical intervention, which should consist of
exposure and direct visualization of potentially infected tissue (including deep soft tissue, fascia, and underlying muscle) and radical resection of affected areas.

Radiologic studies should
be undertaken only in patients in whom the diagnosis is not seriously considered, as they delay surgical intervention and
frequently provide confusing information.

Unfortunately, surgical extirpation of infected tissue frequently entails amputation and/or disfiguring procedures; however, incomplete
procedures arc associated with higher rates of morbidity and mortality.

How well did you know this?
1
Not at all
2
3
4
5
Perfectly
10
Q

What is FALSE regarding intravascular catheter
infections?

A. Selected low-virulence infections can be treated with
a prolonged course of antibiotics.

B. In high-risk patients, prophylactic antibiotics infused through the catheter can reduce rate of catheter infections.

C. Bacteremia with gram-negative bacteria or fungi should prompt catheter removal.

D. Many patients with intravascular catheter infections are asymptomatic.

A

Answer: B

Many patients who develop intravascular catheter infections
are asymptomatic, often exhibiting solely an elevation in the
WBC count.

Blood cultures obtained from a peripheral site
and drawn through the catheter that reveal the presence of the same organism increase the index of suspicion for the presence of a catheter infection.

Obvious purulence at the exit site of the skin tunnel, severe sepsis syndrome due to any type of
organism when other potential causes have been excluded, or
bacteremia due to gram-negative aerobes or fungi should lead to catheter removal.

Selected catheter infections due to low-virulence microbes such as Staphylococcus epidermidis can be effectively treated in approximately 50 to 60% of patients with a 14- to 21-day course of an antibiotic, which should be considered when no other vascular access site exists.

Use of systemic antibacterial or antifungal agents to prevent catheter infection is of no utility and is contraindicated.

How well did you know this?
1
Not at all
2
3
4
5
Perfectly
11
Q

Patients with a penicillin allergy are LEAST likely to have a cross-reaction with

A. Synthetic penicillins

B. Carbapenems

C. Cephalosporins

D. Monobactams

A

Answer: D

Allergy to antimicrobial agents must be considered prior to
prescribing them.

First, it is important to ascertain whether a patient has had any type of allergic reaction in association with administration of a particular antibiotic.

However, one
should take care to ensure that the purported reaction consists of true allergic symptoms and signs, such as urticaria,
bronchospasm, or other similar manifestations, rather than indigestion or nausea.

Penicillin allergy is quite common, the reported incidence ranging from 0.7 to 10%. Although avoiding the use of any beta-lactam drug is appropriate in patients
who manifest significant allergic reactions to penicillins, the incidence of cross-reactivity appears low for all related agents, with 1% cross-reactivity for carbapcncms, 5 to 7%
cross-reactivity for cephalosporins, and extrdtnely small or
nonexistent cross-reactivity for monobactams.

How well did you know this?
1
Not at all
2
3
4
5
Perfectly
12
Q
What is the estimated risk of transmission of human
immunodeficiency virus (HIV) from a needlestick from a source with HIV-infected blood?

A. <0.5%

B. 1%

C. 5%

D. 10%

A

Answer: A

While alarming to contemplate, the risk of human immunodeficiency virus (HIV) transmission from patient to surgeon
is low.

As of May 2011, there had been six cases of surgeons with HIV seroconversion from a possible occupational exposure, with no new cases reported since 1999.

Of the numbers
of health care workers with likely occupationally acquired HIV infection (n = 200), surgeons were one of the lower risk
groups (compared to nurses at 60 cases and nonsurgeon physicians at 19 cases).

The estimated risk of transmission from
a needlestick from a source with HIV-infected blood is estimated at 0.3%.

How well did you know this?
1
Not at all
2
3
4
5
Perfectly
13
Q

Closure of an appendectomy wound in a patient with perforated appendicitis who is receiving appropriate antibiotics will result in a wound infection in what percentage of patients?

A. 3-4%

B. 8-12%

C. 15-18%

D. 22-25%

A

Answer: A

Surgical management of the wound is also a critical determinant of the propensity to develop an SSI.

In healthy individuals, class I and II wounds may be closed primarily, while skin closure of class III and IV wounds is associated with high rates of incisional SSIs (~25-50%).

The superficial aspects of these latter types of wounds should be packed open and allowed to heal by secondary intention, although selective use of delayed primary closure has been associated with a reduction in incisional SSI rates.

It remains to be determined whether National Nosocomial Infections Surveillance (NNIS) system type stratification schemes can be employed prospectively in order to target specific subgroups of patients who will benefit from the use of prophylactic antibiotic and/or specific wound management techniques.

One clear example based on cogent data from clinical trials is that class III wounds in healthy patients undergoing appendectomy for perforated or gangrenous appendicitis can be primarily closed as long as antibiotic therapy directed against aerobes and anaerobes is administered.

This practice leads to SSI rates of approximately 3 to 4%.

How well did you know this?
1
Not at all
2
3
4
5
Perfectly
14
Q

A chronic carrier state occurs with hepatitis C infection

A. 90-99%

B. 75-80%

C. 50-60%

D. 10-30%

A

Answer: B

Hepatitis C virus (HCV), previously known as non-A, non-B hepatitis, is an RNA flavivirus first identified specifically in the late 1980s.

This virus is confined to humans and chimpanzees.

A chronic carrier state develops in 75 to 80% of patients with the infection, with chronic liver disease occurring in three-fourths of patients who develop chronic infection. The number of new infections per year has declined since the 1980s due to routine testing of blood donors for this virus.

Fortunately, HCV is not transmitted efficiently through occupational exposures to blood, with the seroconversion rate after accidental needlestick approximately 1.8%.

How well did you know this?
1
Not at all
2
3
4
5
Perfectly
15
Q

Possible exposure to anthrax should be initially treated with

A. Colistin

B. Ciprofloxacin or doxycycline

C. Amoxicillin

D. Observation

A

Answer: B

Inhalational anthrax develops after a 1- to 6-day incubation period, with nonspecific symptoms including malaise, myalgia, and fever.

Over a short period of time, these symptoms worsen, with development of respiratory distress, chest pain, and diaphoresis.

Characteristic chest roentgenographic findings include a widened mediastinum and pleural effusions.

A key aspect in establishing the diagnosis is eliciting an exposure history.

Rapid antigen tests are currently under development for identification of this gram-positive rod.

Postexposure prophylaxis consists of administration of either ciprofloxacin or doxycycline.

If an isolate is demonstrated to be penicillin-sensitive, the patient should be switched to amoxicillin.

Inhalational exposure followed by the development of symptoms is associated with a high mortality rate.

Treatment options include combination therapy with ciprofloxacin, clindamycin, and rifampin; clindamycin added to blocks production of toxin, while rifampin penetrates into the central nervous system and intracellular locations.

How well did you know this?
1
Not at all
2
3
4
5
Perfectly
16
Q

The most effective postexposure prophylaxis for a surgeon stuck with a needle while operating on an HIV-positive patient is

A. None (no effective treatment is known).

B. Two- or three-drug therapy started within hours of exposure.

C. Single drug therapy started within 24 hours of exposure.

D. Triple drug therapy started within 24 hours of exposure.

A

Answer: B

Postexposure prophylaxis for HIV has significantly decreased the risk of seroconversion for health care workers with occupational exposure to HIV.

Steps to initiate postexposure prophylaxis should be initiated within hours rather than days for the most effective preventive therapy.

Postexposure prophylaxis with a two- or three-drug regimen should be initiated for health care workers with significant exposure to patients with an HIV-positive status.

If a patient’s HIV status is unknown, it may be advisable to begin postexposure prophylaxis while testing is carried out, particularly if the patient is at high risk for infection due to HIV (eg, intravenous narcotic use).

Generally, postexposure prophylaxis is not warranted for exposure to sources with unknown status, such as deceased persons or needles from a sharps container.

How well did you know this?
1
Not at all
2
3
4
5
Perfectly
17
Q

What is NOT an early goal in treatment of severe sepsis?

A. Mean arterial pressure >65 mm Hg

B. Central venous pressure 8 to 12 mm

C. Urine output >0.5 cc/kg/h

D. Serum lactate <2 mmol/L

A

Answer: D

Patients presenting with severe sepsis should receive resuscitation fluids to achieve a central venous pressure target of 8 to 12 mm Hg, with a goal of mean arterial pressure of >65 mm Hg
and urine output of >0.5 cc/kg/h.

Delaying this resuscitative step for as little as 3 hours until arrival in the ICU has been shown to result in poor outcome.

Typically this goal necessitates early placement of central venous catheter.

How well did you know this?
1
Not at all
2
3
4
5
Perfectly
18
Q

A patient in the ICU has been on ventilator support for 3 weeks. He has new onset elevated WBC count, fever, and consolidation seen on chest X-ray.

What is an appropriate next step?

A. Exchange endotracheal tube and change respiratory circuit.

B. Obtain bronchoalveolar lavage.

C. Start treatment with empiric penicillin G.

D. Obtain chest CT.

A

Answer: B

Prolonged mechanical ventilation is associated with nosocomial pneumonia.

These patients present with more severe disease, are more likely to be infected with drug-resistant pathogens, and suffer increased mortality compared with patients who develop community-acquired pneumonia.

The diagnosis of pneumonia is established by presence of a purulent sputum, elevated leukocyte count, fever, and new chest X-ray abnormalities such as consolidation.

The presence of two of the clinical findings, plus chest X-ray findings, significantly increases the likelihood of pneumonia.

Consideration should be given to performing bronchoalveolar lavage to obtain samples for Gram stain and culture.

Some authors advocate quantitative cultures as a means to identify a threshold for diagnosis.

Surgical patients should be weaned from mechanical ventilation as soon as feasible, based on oxygenation and inspiratory effort, as prolonged mechanical ventilation increases the risk of nosocomial pneumonia.

How well did you know this?
1
Not at all
2
3
4
5
Perfectly
19
Q

Patients with severe, necrotizing pancreatitis should be treated with

A. No antibiotics unless CT-guided aspiration of the area yields positive cultures

B. Empiric cefoxitin or cefotetan

C. Empiric cefuroxime plus gentamicin

D. Empiric carbapenems or fluoroquinolones

A

Answer: D

Current care of patients with severe acute pancreatitis includes staging with dynamic, contrast-enhanced helical CT scan with 3-mm tomographs to determine the extent of
pancreatic necrosis, coupled with the use of one of several prognostic scoring systems.

Patients who exhibit significant pancreatic necrosis should be carefully monitored in the ICU and undergo follow-up CT examination. The weight of current evidence also favors administration of empiric antibiotic therapy to reduce the incidence and severity of secondary
pancreatic infection, which typically occurs several weeks after the initial episode of pancreatitis.

Several randomized, prospective trials have demonstrated a decrease in the rate of
infection and mortality using agents such as carbapenems or
fluoroquinolones that achieve high pancreatic tissue levels.

How well did you know this?
1
Not at all
2
3
4
5
Perfectly
20
Q

A patient with a localized wound infection after surgery should be treated with

A. Antibiotics and warm soaks to the wound

B. Antibiotics alone

C. Antibiotics and opening the wound

D. Incision and drainage alone

A

Answer: D

Effective therapy for incisional SSIs consists solely of incision and drainage without the addition of antibiotics.

Antibiotic therapy is reserved for patients in whom evidence of severe
cellulitis is present, or who manifest concurrent sepsis syndrome.

The open wound often is allowed to heal by secondary intention, with dressings being changed twice a day.

The use of topical antibiotics and antiseptics, to further wound healing, remains unproven, although anecdotal studies indicate their potential utility in complex wounds that do not heal with
routine measures.

How well did you know this?
1
Not at all
2
3
4
5
Perfectly
21
Q

Which areas likely do NOT contain resident
microorganisms?

A. Terminal ileum

B. Oropharynx

C. Main pancreatic duct

D. Nares

A

Answer: C

The urogenital, biliary, pancreatic ductal, and distal respiratory tracts do not possess resident microflora in healthy
individuals, although microbes may be present if these barriers are affected by disease (eg, malignancy, inflammation,
calculi, or foreign body), or if microorganisms are introduced from an external source (eg, urinary catheter or pulmonary aspiration).

In contrast, significant numbers of microbes are
encountered in many portions of the gastrointestinal tract,
with vast numbers being found within the oropharynx and
distal colorectum, although the specific organisms differ.

How well did you know this?
1
Not at all
2
3
4
5
Perfectly
22
Q

On the eighth day after an exploratory laparotomy and bowel resection complicated by intraabdominal hypertension, a 65-year-old female who remains intubated in the intensive care unit (ICU) develops a fever of 102°F. An infectious workup reveals a new right lower lobe consolidation. When initiating antibiotic therapy for presumed ventilator-associ- ated pneumonia (VAP), which of the following does not treat Pseudomonas aeruginosa?

A. Cefepime

B. Unasyn (ampicillin/sulbactam)

C. Ticarcillin

D. Aztreonam

E. Ciprofloxacin

A

ANSWER: B

P. aeruginosa is a gram-negative bacillus commonly implicated in VAP.

Antipseudomonal antibiotics should be initiated empirically in any patient with VAP prior to isolation of the organism on culture due to the high mortality associated with pseudomonal infection.

Antipseudomonal penicillins include ticarcillin and piper- acillin. Third- and fourth-generation cephalosporins, such as ceftazidime and cefepime, are effective against P. aeruginosa and have a relatively narrow range of activity, making them preferred agents in susceptible isolates.

Monobactams (like aztreonam) and carbapenems (meropenem, imipenem) are effective, but have a very broad spectrum of activity, and should be deescalated once susceptibilities are available.

Fluoroquinolones are also effective. The polymyxin colistin is also effective, but has an extensive toxicity profile, and should be used cautiously with multiresistant organisms.

How well did you know this?
1
Not at all
2
3
4
5
Perfectly
23
Q

A 67-year-old male remains in the hospital 1 week after undergoing a pancreaticoduodenectomy.

He has two intraabdominal closed-suction drains in place, as well as a left internal jugular triple lumen catheter; his Foley catheter was removed on the third postoperative day.

On the seventh postoperative day, he becomes febrile to 101.5°F, and a fever workup reveals a growth of Enterococcus in two of two peripheral blood cultures.

Which of the following is true regarding the diagnosis of a central line–associated bloodstream infection (CLABSI)?

A. It is preferential to begin empiric antimicrobial therapy prior to obtaining cultures.

B. Catheter-site exudate, if present, should not be cultured when there is concern for a line-related bloodstream infection.

C. The subcutaneous portion of the central venous catheter should be cultured, rather than the tip.

D. Paired blood samples (one from the catheter and one from a peripheral vein, or alternatively from greater than two lumens of the same central venous catheter) growing the same organism at levels that meet catheter-related bloodstream infection criteria are required to diagnose a CLABSI.

E. Growth of greater than 10 colony-forming units (cfu) by semiquantitative (roll-plate) culture confirms catheter colonization.

A

D

How well did you know this?
1
Not at all
2
3
4
5
Perfectly
24
Q

Which of the following is true regarding the treatment of catheter-related bloodstream infections?

A. All catheters in cases of confirmed CLABSI should be removed; it is never appropriate to attempt to salvage the infected catheter.

B. Empiric coverage of Candida should be initiated in bone marrow or solid organ transplant patients with presumed CLABSI.

C. Empiric antibiotic therapy should include methicillin- resistant Staphylococcus aureus (MRSA) coverage as well as gram-negative rod (GNR) coverage, regardless of the severity of illness.

D. Duration of antibiotic therapy in CLABSIs is timed from the day when empiric antibiotics were initiated.

E. The location of a temporary central venous catheter (subclavian versus internal jugular versus femoral) has no influence on the empiric antibiotic agents that should be used.

A

B

COMMENTS: Central venous catheters are commonly used in many settings in modern health care, but their use is associated with the risk of bloodstream infections, known as CLABSIs. These infections are known to increase morbidity, mortality, and health care costs.

To diagnose a CLABSI, growth of greater than 15 cfu by semiquantitative (roll-plate) culture from at least two samples is required; these two samples may be obtained from the catheter and a peripheral vein or, alternatively, from at least two lumens from the same central venous catheter.

The diagnosis of CLABSI is best defined by a colony count threefold greater than that obtained from a peripheral vein. Skin and catheter hubs should be prepared with alcohol, tincture of iodine, or alcohol-based chlorhexidine (>0.5%) with adequate drying time, prior to obtaining cultures; cultures should be obtained by trained phlebotomists if possible.

Management of CLABSI varies based on the organism cultured and the severity of illness, but for all CLABSIs, the duration of antimicrobial therapy is determined on the first day of obtaining negative blood cultures.

Preferably, adequate cultures are obtained prior to the initiation of antibiotic therapy.

Empiric therapy in uncomplicated cases [i.e., cases without evidence of severe sepsis, endocarditis or osteomyelitis (OM), or without evidence of infection of the catheter tunnel or adjacent abscess] should begin with antibiotics that cover gram-positive cocci.

Vancomycin or daptomycin for empiric therapy should be reserved for areas with a high prevalence of MRSA. Empiric GNR coverage should be added in cases of severe sepsis, neutropenia, in patients with known colonization with a GNR organism, or in patients with femoral catheters.

Empiric coverage for Candida, either with an echinocandin or a fluconazole, should be initiated in patients with severe sepsis plus prolonged broad-spectrum antibiotic use, total parenteral nutrition, hematologic malignancy or receipt of solid organ or bone marrow transplants, or known Candida colonization.

In most circumstances, the infected catheter should be removed; however, there are certain instances in which salvage of the catheter may be attempted.

Antibiotic locks (antibiotic solutions that are instilled into the catheter itself) can be used in conjunction with systemic antimicrobial therapy, particularly in patients in whom catheters are difficult to remove or replace (i.e., tunneled hemodialysis catheters or ports for parenteral nutrition in short gut syndrome).

If patients have persistent positive blood cultures after salvage attempt, the catheter should be removed.

How well did you know this?
1
Not at all
2
3
4
5
Perfectly
25
Q

A 78-year-old man with a history of urinary retention and a chronic indwelling urinary catheter is admitted to the hospital from his nursing home with a new-onset altered mental status, and a catheter-associated urinary tract infection (CAUTI) is suspected. Which of the following is true regarding CAUTIs?

A. A 7-day antibiotic treatment is adequate for patients whose symptoms respond promptly to treatment.

B. A CAUTI can be sufficiently diagnosed by the presence of greater than 105 cfu/mL of at least one bacterial species in a urine specimen.

C. Urine specimens being sent for culture can be obtained from the catheter bag.

D. Pyuria is a specific indicator for urinary tract infections (UTIs).

E. Proteus mirabilis is the most common organism cultured in CAUTIs.

A

A

COMMENTS: CAUTIs are the most common health care–associ- ated infection worldwide, and the most important factor leading to nosocomial UTIs is urinary catheterization.

The best prevention for CAUTIs is avoiding catheterization. There are a limited number of circumstances in which catheterization is appropriate, such as when monitoring urine output in critically ill patients, in patients with acute urinary retention or obstruction, in certain surgical procedures, or to facilitate healing of wounds or pressure ulcers in some patients with urinary incontinence.

A CAUTI is diagnosed by the presence of greater than 103 cfu/mL of at least one bacterial species in a catheter urine specimen or a midstream-voided urine specimen in addition to clinical signs and symptoms suggestive of infection. Signs and symptoms of a UTI include new-onset fever, rigors, altered mental status, lethargy, malaise, flank pain, costovertebral angle tenderness, hematuria, suprapubic or pelvic discomfort, dysuria, urinary frequency, and urinary urgency.

For these symptoms to be attributed to a catheter, the patient must have a current indwelling urinary catheter or have had one within the 48 h preceding his or her symptoms. Without these symptoms, an infection cannot be diagnosed.

Catheter-associated asymptomatic bacteriuria is more likely, and this is diagnosed by the presence of greater than 105 cfu/mL of at least one bacterial species in a urine specimen.

Catheters predispose to bacteriuria and UTIs in a variety of ways, but formation of a biofilm along the catheter itself is the most important predisposing factor.

Pyuria is not specific for UTIs; it can be seen in a variety of other renal pathologies and should not be used as a diagnostic criterion for UTI.

Urine culture specimens are best collected by removing the catheter, if possible, to obtain a voided midstream specimen, from the tubing or catheter itself in a catheter that has been in place for less than 2 weeks, or by removing any catheter that has been in place for greater than 2 weeks and obtaining a specimen from the new catheter.

The duration of antibiotic therapy should be 7 days for patients whose symptoms respond promptly to antibiotics and 10 to 14 days for those whose symptoms respond slowly.

Escherichia coli is the most common causative organism in CAUTIs, although Proteus is commonly cultured in patients with chronic indwelling catheters.

How well did you know this?
1
Not at all
2
3
4
5
Perfectly
26
Q

A 57-year-old Asian American female presents to her hepatologist’s office for monitoring of her known chronic hepatitis B infection. Which of the following sets of test results is consistent with chronic active hepatitis B infection?

A. Hepatitis B surface antigen (HBsAg)+ less than 6 months, hepatitis B surface antibody (HBsAb)−, immunoglobulin M (IgM) anti-HBc+, elevated aspartate transaminase (AST), and alanine transaminase (ALT)

B. HBsAg+ greater than 6 months, HBsAb−, HBcAb+, hepatitis B virus (HBV) DNA > 20,000 IU/mL, mildly elevated AST and ALT

C. HBsAg+ greater than 6 months, hepatitis B e antigen (HBeAg)−, HBV DNA < 2000 IU/mL, normal AST and ALT

D. HBsAg−, HBsAb+, HBcAb+, normal AST and ALT

E. HBsAg−, HBsAb+, HBcAb−, normal AST and ALT

A

ANSWER: B

COMMENTS: Answer A is acute hepatitis B infection; chronic infection requires HBsAg positivity for at least 6 months.

A patient with chronic active infection (answer B) exhibits normal to mildly elevated liver enzymes and HBsAg positivity but negative HBsAb since the infection has not been cleared; HBcAb will be positive with chronic infection, and HBeAg may be positive as well if there is a continued high level of viral replication; this is usually accompanied by a high level of HBV DNA.

This is different from an inactive carrier state (answer C); these patients have persistent HBV infection of the liver without significant hepatic necrosis or inflammation, so their liver enzymes are not significantly elevated; there is a low level of viral replication, which correlates with negative HBeAg.

Patients who have cleared HBV infection have evidence of HBsAb and HBcAb positivity (answer D).

Answer E reflects successful vaccination, with only HBsAb positivity on serologic testing.

How well did you know this?
1
Not at all
2
3
4
5
Perfectly
27
Q

Match the antibiotic and its classical toxicity profile.

A. Vancomycin

B. Aminoglycosides

C. Isoniazid (INH)

D. Fluoroquinolones

E. Tetracycline

a. Tendinopathy
b. Red man syndrome
c. Phototoxicity
d. Hepatitis
e. Ototoxicity

A

A-b, B-e, C-d, D-a, E-c

COMMENTS: Vancomycin is known to cause red man syndrome, a syndrome composed of flushing of the face, neck, and chest.

It is better described as a hypersensitivity reaction, rather than a true allergy, because the effect is partly mediated by the speed with which it is transfused.

Aminoglycosides can cause ototoxicity (cochlear and vestibular), which is dose dependent.

The effects may begin to be seen even after cessation of the drug; aminoglycoside ototoxicity may be irreversible.

INH can cause severe, sometimes fulminant, hepatitis that is largely indistinguishable from acute viral hepatitis.

The mechanism of toxicity is not clear, but is thought to be related to direct toxicity of the drug or its metabolites, and is more likely to occur when other hepatitis risk factors are present, such as concurrent alcohol consumption, use of other drugs that utilize the cytochrome P450 system for metabolism, previous INH intolerance, or prior or concurrent liver disease.

Fluoroquinolone- induced tendinopathy is rare, but it has been documented with almost all drugs in this class.

Tetracyclines cause cutaneous photo- toxicity, so patients taking tetracyclines are cautioned to avoid sun exposure.

How well did you know this?
1
Not at all
2
3
4
5
Perfectly
28
Q

Which of the following is not a Surgical Care Improvement Project (SCIP) measure for infection prevention in surgical patients?

A. The optimal timing for administration of prophylactic antibiotics is within 1 h of surgical incision.

B. Prophylactic antibiotics should be discontinued within 24 h of the end of surgery; in cardiac surgery, this is lengthened to 48 h.

C. Clippers are preferred to razors for preoperative hair removal, if necessary.

D. Goal blood glucose in the first 48 h following surgery is less than 160 mg/dL.

E. Patients should remain normothermic within the first hour following surgery.

A

ANSWER: D

COMMENTS: The SCIP summarizes specific tactics aimed at prevention of surgical site infections (SSIs).

Of the answer choices listed above, only D is inaccurate; optimal blood glucose within the first 48 h of surgery is less than 200 mg/dL.

Hyperglycemia impairs the host immune function and is known to increase the risk of infection in both diabetic and nondiabetic patients.

Moderate hyperglycemia (i.e., blood glucose > 200 mg/dL) in the first 24 h following surgery increases the risk of SSIs by a factor of four.

Tight blood glucose control has been a matter of debate in recent years, with some arguing that very strict blood glucose control (i.e., less than 110 mg/dL) results in significantly decreased rates of infection.

However, postoperative hypoglycemia is associated with increased mortality, so glycemic goals have been relaxed.

The remaining answer choices are correct. Prophylactic antibiotics should be given within 1h of incision, though 2h is appropriate for fluoroquinolones and vancomycin, due to the prolonged infusion times for these drugs.

They should be discontinued within 24 h of the end of surgery in all cases aside from the cardiac surgery, where 48 h of prophylactic antibiotic therapy is appropriate.

Razors should never be used to remove hair prior to procedures due to the increased risk for small breaks in the skin, which might introduce infection; clippers should be used preoperatively.

Normothermia, defined as any temperature between 96.8°F and 100.4°F, should be maintained intraoperatively and for at least the first hour following surgery.

How well did you know this?
1
Not at all
2
3
4
5
Perfectly
29
Q

Match the surgical procedure with the most appropriate preoperative prophylactic antibiotic.

A. Elective laparoscopic cholecystectomy

B. Femoral to popliteal arterial bypass with graft

C. Cystoscopy with ureteral stent placement

D. Right hemicolectomy

E. Parotidectomy

a. Ertapenem
b. Clindamycin
c. None
d. Cefazolin
e. Ciprofloxacin

A

A-c, B-d, C-e, D-a, E-b

COMMENTS: Prophylactic antibiotics administered preoperatively should be targeted to the organisms most likely to be encountered in the operative field.

Broad-spectrum antibiotics do not have greater efficacy at preventing SSIs than more narrow-spectrum, targeted choices.

The choice of prophylactic antibiotic therapy for intraabdominal surgeries varies widely depending on the exact location within the gastrointestinal tract that is being manipulated. Low-risk biliary tract procedures (e.g., elective laparoscopic cholecystectomy) do not require surgical site infection prophylaxis; however, patients undergoing open or complicated procedures involving the biliary tract should receive antibiotics covering enteric GNRs, Enterococcus, and Clostridia.

Comparatively, colorectal surgery requires broad coverage of enteric GNRs, anaerobes, and Enterococcus, which may be accomplished with ertapenem, a carbapenem antibiotic.

Cystoscopy with manipulation, such as the placement of ureteral stents, necessitates coverage of enteric GNRs and Enterococcus; compared with colorectal surgery, anaerobic coverage is not necessary.

Vascular SSIs are most commonly caused by skin flora, such as Staphylococcus and Streptococcus species, so a first-generation cephalosporin, such as cefazolin, is adequate.

In clean-contaminated head and neck cases (i.e., any surgical procedure involving the oropharyngeal mucosa), prophylactic antibiotics should cover both aerobic and anaerobic oral flora (such as Streptococcus, Bacteroides, and Peptostreptococcus).

How well did you know this?
1
Not at all
2
3
4
5
Perfectly
30
Q

A 68-year-old female has been admitted to the emergency room with recurrent Clostridium difficile colitis. Her first episode of C. difficile colitis was 3 months prior after receiving clindamycin for a mild episode of cellulitis. Two months ago, she had a second episode, treated again with full symptom resolution. On examination, her vital signs are normal and her abdomen is benign, with only mildly tender to deep palpation in the right lower quadrant. Laboratory results are notable for a leukocytosis of 13.4,000 cells per MCL with 86% neutrophils, mild hypokalemia, and positive C. difficile stool antigen. An abdominal film shows a colon of normal caliber. What is the most appropriate treatment for this patient?

A. Oral metronidazole 500 mg every 8 h for 10 to 14 days

B. Intravenous (IV) metronidazole 500 mg every 8 h for 10 to 14 days

C. Oral vancomycin 125 mg every 6 h for 10 to 14 days

D. Oral vancomycin, in a tapered and pulsed fashion over approximately 5 to 7 weeks

E. IV vancomycin 125 mg every 6 h for 10 to 14 days

A

D

How well did you know this?
1
Not at all
2
3
4
5
Perfectly
31
Q

The patient in the question above is admitted and started on antibiotics, but her condition continues to deteriorate clinically over the next 2 days despite appropriate antibiotic therapy, probiotics, and supportive treatment. Her white blood cell (WBC) count continues to rise to 17.8 and creatinine increases to 1.5 from baseline of 0.8. She has a low-grade fever and marginal urine output, and her abdomen becomes distended, tympanic, and tender. An abdominal obstructive series shows dilation of the entire colon to 10 cm in diameter without evidence of pneumoperitoneum. Which of the following is not an acceptable course of action in treating severe C. difficile colitis?

A. Transitioning from pulsed to standard scheduled oral vancomycin 125 mg every 6 h with the addition of IV metronidazole 500 mg every 8 h

B. Transitioning from pulsed to standard scheduled oral vancomycin 125 mg every 6 h with the addition of oral fidaxomicin 200 mg every 12 h

C. Subtotal colectomy

D. Diverting loop ileostomy with colonic lavage

E. Metronidazole enemas

A

E

How well did you know this?
1
Not at all
2
3
4
5
Perfectly
32
Q

Which of the following is true regarding the pathophysiology
of C. difficile infection?

A. Antimicrobial agents with activity against C. difficile are equally as likely to result in C. difficile colitis as those without activity against C. difficile.

B. A patient’s inability to produce antibody to toxin A is a significant predictor of recurrent C. difficile infection.

C. Advanced age is not considered a risk factor for develop- ment of clinical C. difficile infection.

D. Studies have suggested that gastric acid suppression [i.e., use of proton pump inhibitors (PPIs) or H2 blockers] may be protective against the development of C. difficile infection.

E. Alcohol-based hand sanitizers are effective in removing C. difficile spores after contact with an infected patient.

A

ANSWER: B

Recurrent C. difficile colitis is a growing health problem in the United States.

Treatment of an initial episode can be accomplished with oral or IV metronidazole (500 mg every 8 h for 10 to 14 days) or oral vancomycin (125 mg every 6 h for 10 to 14 days).

IV vancomycin is never acceptable as a treatment for C. difficile colitis. More severe initial infections should be treated with vancomycin rather than metronidazole.

For a patient’s first relapse, if there is no evidence of systemic toxicity, treatment with the initial antibiotic regimen may be appropriate.

Second relapses and beyond, however, require oral vancomycin in a tapered and pulsed fashion (see the following chart).

Alternatively, fidaxomicin, a macrolytic antibiotic that is bactericidal against C. difficile, can be used.

With both drugs, probiotics may be added, though their efficacy is still unclear.

Fecal microbiota transplant has also been shown to be a cost-effective solution for recurrent C. difficile colitis.

For patients developing evidence of systemic toxicity from C. difficile colitis, more aggressive therapy is required.

Depending on the patient’s clinical stability, multiple avenues of treatment are available.

Scheduled oral vancomycin or fidaxomicin can be given, with or without IV metronidazole; IV metronidazole may have enhanced efficacy in patients with evidence of bowel dysmotility.

The duration of antibiotics in severe C. difficile colitis is generally at least 17 to 24 days (1 week beyond standard treatment). Intra- colonic vancomycin administration (i.e., vancomycin enema) is also an acceptable treatment method; metronidazole enema is not an accepted treatment, which makes answer E incorrect.

Vancomycin enemas are particularly useful in patients who have conditions preventing oral vancomycin from reaching the colon (i.e., end ileostomies, severe ileus, colonic dysmotility, etc.).

Patients who are severely ill with toxic megacolon, perforation, uncontrolled sepsis, or multiorgan failure should be considered operative candi- dates.

The two most accepted surgical procedures are subtotal colectomy and diverting loop ileostomy with colonic lavage followed by antegrade vancomycin enemas.

The pathogenesis of C. difficile infection rests largely on the disruption of normal colonic flora by other antibiotics. Those with inherent activity against C. difficile (i.e., those who have a robust antibody response to toxin A) are less likely to become clinically infected.

One study reported that patients who did not develop antibodies to toxin A during their initial infection were 48 times as likely to develop a recurrent infection.

Advanced age (>65 years) is predictive of both initial and recurrent infection.

Additional patient-specific risk factors for infection include gastric acid suppression (either via PPIs or H2 blockers), recent gastrointestinal surgery, chemotherapy, stem cell transplant, and obesity.

The three classes of antibiotics cited most frequently as causative agents are clindamycin, cephalosporins, and fluoroquinolones, and these agents are generally administered weeks to several months prior to the development of C. difficile infection.

Concomitant use of multiple antibiotics and prolonged courses of antibiotics have also been found to be risk factors for infection; however, not all patients who are exposed to C. difficile and receive antibiotics develop an infection.

Infection control policies are paramount to controlling the nosocomial spread of C. difficile infection; patients with active infections should be placed on contact precautions, and all healthcare workers (HCWs) who encounter the patient should wash their hands with soap and water since C. difficile spores are resistant to alcohol-based hand sanitizers.

Tapered and Pulsed Vancomycin Dosing for Recurrent C. difficile Infection:
Tapered dosing: 125 mg four times daily for 10 to 14 days, followed by 125 mg twice daily for 7 days, and then 125 mg daily for 7 days
Pulse dosing (after completion of the taper): 125 mg every 2 to 3 days for 2 to 8 weeks
How well did you know this?
1
Not at all
2
3
4
5
Perfectly
33
Q

Which of the following is true regarding postoperative fever?

A. Urinalysis, urine culture, and chest x-ray must be obtained as part of a complete fever workup in postopera- tive patients within 72 h of operation.

B. In a febrile postoperative patient, wound cultures should be obtained regardless of the appearance of the wound.

C. Wound infections in the first 24 to 48 h are uncommon but, if present, are worrisome for group A streptococcal or clostridial infection.

D. Fevers persisting for greater than 96 h postoperatively are expected in cases of diffuse intraabdominal infection, such as feculent peritonitis from diverticulitis, even with appropriate surgical management.

E. All of the above.

A

ANSWER: C

COMMENTS: Fever is a common postsurgical finding, but not all fevers require evaluation.

In the first 72 h after surgery, fever is more likely related to postoperative inflammatory responses in the host, rather than infection, when the patient is otherwise asymptomatic or without an indwelling urinary catheter.

While fever is expected in postoperative patients with evidence of diffuse infection preoperatively (such as large abscesses, purulent or feculent peritonitis, or necrotizing soft tissue infection), fevers that are persistent for greater than 96 h after appropriate surgical management, or recurrent fevers after 96 h, are concerning for recurrent or incompletely cleared infection.

Fevers presenting greater than 96 h post-operatively are more likely to be due to infection.

Surgical wounds should only be cultured if there are signs or symptoms suggestive of wound infection (i.e., purulence, tenderness, and erythema). SSIs are most commonly due to the native flora of the organ that was operated upon, but within the first 24 to 48 h of surgery, wound infections are more likely to be due to group A Streptococcus (GAS) or Clostridia, which are significantly more virulent organisms.

Deep venous thrombosis, pulmonary embolism, and superficial thrombophlebitis are other possible etiologies that should be considered in the evaluation of a febrile postoperative patient.

34
Q

A 25-year-old male remains nasotracheally intubated in the ICU while undergoing repeated debridements for Ludwig’s angina. His infection appears to be adequately drained, although he still has persistent facial swelling. Sinusitis is expected. Which of the following is true regarding sinusitis in critically ill patients?

A. The most important risk factor for sinusitis in critically ill patients is a history of MRSA of the nares.

B. Few sinus infections are polymicrobial; often, only one organism is isolated in culture.

C. The most commonly isolated organism in sinusitis cultures is coagulase-negative S. aureus.

D. Computed tomography (CT) scan of the sinuses is the most sensitive diagnostic imaging modality for sinusitis.

E. Incidence of acute sinusitis in nasotracheally intubated patients is approximately 75% after 1 week.

A

ANSWER: D

COMMENTS: Sinusitis is an infrequent but potentially life-threatening cause of fever in critically ill patients.

When the ostia of the sinuses become obstructed (e.g., from nasotracheal intubation, nasogastric tube placement, or maxillofacial trauma), bacterial overgrowth can occur, and when drainage is impaired, infection results; approximately one-third of patients who are nasotracheally intubated develop sinusitis within 1 week.

The diagnosis is suggested by clinical findings, including cough, purulent nasal dis- charge, tooth pain, fever, wheezing, or throat or tooth pain.

Normal nasopharyngeal organisms are the most common causes, and infec- tions are often polymicrobial.

GNRs are present in approximately two-third of cases of acute sinusitis in critically ill patients; gram- positive rods are isolated in approximately one-third of cases, and fungi are found in 5%–10%.

A CT scan of the sinuses is the most sensitive imaging modality for diagnosing sinusitis, although plain films can be obtained as well.

Plain films are less sensitive than CT scan, but the accuracy of plain films in diagnosis can be augmented if combined with the findings of nasal endoscopy, when performed by a skilled endoscopist. Needle aspiration of the sinuses to obtain fluid for cultures provides a definitive diagnosis, but a biopsy may be required to rule out invasive fungal sinusitis in immunocompromised patients.

35
Q

A 24-year-old male presents to the emergency department after sustaining a puncture wound to his left foot 60 min prior to presentation. On examination, he has a small metal nail protruding from the plantar aspect of his left foot, with moderate surrounding erythema and a small amount of bleeding, but no significant purulence. He is unsure of his tetanus vaccination status. How should the issue of potential tetanus infection be addressed in this patient?

A. Local wound care only

B. Local wound care, IV metronidazole or penicillin for 7 to 10 days

C. Local wound care, IV metronidazole or penicillin for 7 to 10 days, tetanus toxoid

D. Local wound care, IV metronidazole or penicillin for 7 to 10 days, tetanus toxoid, tetanus immunoglobulin

E. No treatment

A

D

36
Q

Which of the following is true regarding tetanus infection?

A. Tetanus is caused by tetanus toxin, which is produced by
C. tetani, an aerobic gram-positive bacillus.

B. Tetanus infection has purely motor neuron effects.

C. Tetanus-prone wounds include contaminated wounds (i.e., soil, saliva, and stool), crush wounds, or burn wounds.

D. Tetanus immunoglobulin is indicated in any patient with an unknown tetanus vaccination history.

E. None of the above.

A

ANSWER: C

COMMENTS: Tetanus toxin, produced by C. tetani, an anaerobic gram-positive bacillus, causes disinhibition of lower motor neurons; it is taken up via lower motor neurons and transported proximally to the spinal cord and brainstem and then into inhibitory neurons, causing a functional denervation of motor neurons.

This can result in lockjaw; muscle rigidity; and spasm of respiratory, laryngeal, and abdominal muscles, causing respiratory failure.

Autonomic dysfunction can also be seen with tetanus, manifesting as labile blood pressure and heart rate.

Tetanus- prone wounds include contaminated wounds (i.e., soil, saliva, and stool), crush wounds, or burn wounds.

Treatment of tetanus- prone wounds includes local wound care (i.e., debridement and irrigation) and antibiotic therapy targeted against C. tetani.

Depending on the completeness of a patient’s tetanus vaccination, additional therapy is required, based on the following tables:

TETANUS PRONE WOUNDS
Previous Tetanus Toxoid Administration:
1) Less than three doses
- Tetanus Toxoid
- Tetanus Immunoglobulin

2) Three or more doses
- Tetanus Toxoid IF the last dose was prior to >5 years

CLEAN WOUNDS
Previous Tetanus Toxoid Administration:
1) Less than three doses
- Tetanus Toxoid

2) Three or more doses
- Tetanus Toxoid IF the last dose was prior to >10 years

Tetanus immunoglobulin serves to bind tetanus toxin in the bloodstream to minimize absorption into motor neurons. Muscle spasm and rigidity should be treated if present.

37
Q

A 62-year-old man with a history of chronic pancreatitis from alcohol abuse presents to the emergency department with complaints of fevers and abdominal pain. On examination, he is febrile to 100.8°F. He is jaundiced and has right upper quadrant tenderness. His liver enzymes are elevated with a new leukocytosis. A CT scan of the abdomen demonstrates cirrhotic liver morphology, calcifications along the pancreas, and a rim-enhancing hypoechoic liver lesion. Which of the following is true regarding the diagnosis of a pyogenic liver abscess (PLA)?

A. Most PLAs are found in the left hepatic lobe.

B. The most common etiology of PLAs is seeding from another intraabdominal infection via the portal vein.

C. Most PLAs are polymicrobial, and in the United States, E. coli is the most commonly isolated organism.

D. Percutaneous drainage is recommended for all PLAs if the drainage is technically feasible.

E. The classic triad of fever, right upper quadrant pain, and malaise is present in many patients with PLAs.

A

ANSWER: C

COMMENTS: PLAs are relatively uncommon and have vague presenting symptoms. The classic triad of fever, right upper quad- rant pain, and malaise is present in only about one-third of patients.

PLAs have multiple causes including biliary tract disease causing obstruction, seeding from another intraabdominal infection via the portal vein, hepatic artery seeding or bacteremia, direct extension from an adjacent infection (such as cholecystitis and perinephric abscess), or trauma; a small percentage of cases are cryptogenic.

Biliary tract disease is now the most common etiology of PLAs and includes cases of choledocholithiasis, biliary stricture, congenital anomalies of the biliary tree such as choledochal cysts, and obstructing tumors like cholangiocarcinoma or pancreatic adenocarcinoma.

Most infections are polymicrobial, and E. coli is the most frequently isolated organism in the United States, followed by Kleb- siella pneumoniae.

PLAs occur most frequently in the right lobe of the liver, given its greater portal venous flow. Treatment involves broad-spectrum antibiotics (though particularly targeted against enteric GNRs and anaerobes) with percutaneous drainage for patients who do not improve with 2 to 3 days of appropriate anti-microbial therapy, abscesses greater than 5 cm in diameter, or abscesses at risk of rupture.

Surgical intervention is generally reserved for ruptured abscesses, failed response to percutaneous drainage, uncorrected primary pathology (e.g., biliary stricture), or multiloculated abscesses not amenable to percutaneous drainage.

38
Q

A 47-year-old man has been admitted to the burn ICU for 4 days after sustaining a 30% total body surface area (TBSA) burn, involving the neck, chest, bilateral upper extremities, and lower extremities, when he begins to clinically decompensate. Which of the following is suggestive or diagnostic of a burn wound sepsis?

A. Conversion of a partial-thickness burn to a full-thickness burn

B. Fevers greater than 100.4°F

C. Burn wound culture swab growing greater than 105 organisms per gram of tissue swabbed

D. Inability to tolerate enteral tube feeds for greater than 24 h

E. Failure to improve with broad-spectrum antibiotic administration

A

ANSWER: A

COMMENTS: Burn wound infections remain a major cause of morbidity and mortality in burn patients, especially in those with greater than 20% TBSA burns.

Burn wounds have a greater propensity for infection than the healthy tissue since the wound lacks a barrier to environmental organisms, and devascularized burn tissue provides a protein-rich medium in which bacteria can thrive; in addition, burns cause significant systemic responses that inhibit a patient’s normal host defense and healing mechanisms.

Most burns are colonized with bacteria early after injury but do not cause systemic symptoms.

Topical antimicrobials help to reduce the rate of conversion into invasive wound infections; however, invasive infections do result in systemic symptoms and must be treated.

Diagnosis of a burn wound infection requires positive histopathology: the burn must be biopsied, with the growth of >105 organisms per gram of burn wound tissue.

Burn wound infections often mani- fest with physical changes in the appearance of the burn, such as conversion to a greater degree of burn and surrounding cellulitis.

Because most burn patients exhibit the signs of a systemic inflammatory response syndrome (SIRS), the American Burn Association has developed criteria for specifically diagnosing burn wound sepsis:

At least one of the following:
• Culture-positive infection (i.e., blood, urine, wound)
• Pathologic tissue source identified (>105 bacteria per gram
of tissue on quantitative biopsy)
• Clinical improvement with antimicrobial therapy

At least three of the following:
• Temperature greater than 102.2°F or less than 97.7°F
• Progressive tachycardia
• Progressive tachypnea
• Thrombocytopenia that occurs greater than 3 days after
initial resuscitation
• Refractory hypotension
• Leukocytosis > 12,000 or <4000 WBCs/cells MCL
• Hyperglycemia
• Inability to tolerate enteral feedings for >24 h

39
Q

Which of the following is not a classical clinical manifesta-
tion of GAS infection?

A. Necrotizing soft tissue infection

B. Acute rheumatic fever

C. Toxic shock syndrome

D. Pharyngitis

E. Meningitis

A

ANSWER: E

COMMENTS: GAS, or group A S. pyogenes, is an aerobic gram-positive coccus that is responsible for a wide range of clini- cal illnesses, accounting for greater than 600 million infections globally each year.

The infections caused by GAS vary in clinical severity, from mild cellulitis to life-threatening toxic shock syndrome, as demonstrated by the answer choices above.

GAS is also responsible for a variety of postinfectious immune-mediated diseases, such as acute rheumatic fever or poststreptococcal glomerulonephritis.

All the answer choices above can be classically attributed to GAS infection except for meningitis, which is more commonly caused by group B Streptococcus, particularly in newborns.

GAS, which colonizes epithelial surfaces, is almost universally sensitive to penicillin.

Its main virulence factor is the M protein, which is a surface antigen and virulence factor encoded by the emm gene, of which there are over 200 types.

Different emm types are associated with different types of infection; some are known to cause cutaneous infection, while others cause invasive infection.

The prevalence of different emm types is also socioeconomically distinct, with certain emm types being seen in more industrialized areas compared with other types being prevalent in more rural communities.

Geography plays a role as well, with variability in the prevalence of emm types based on region.

40
Q

A 45-year-old man who underwent a splenectomy for immune thrombocytopenic purpura develops a fever. Which of the following is true regarding postsplenectomy sepsis?

A. A fever without other localizing symptoms of infection (such as cough and diarrhea) is usually not worrisome in post-splenectomy patients.

B. Postsplenectomy sepsis is almost never seen in patients who complete the appropriate vaccinations.

C. The most common organism implicated in postsplenec- tomy sepsis is Haemophilus influenzae.

D. Initiation of empiric antibiotics should be delayed until cultures are obtained.

E. Ceftriaxone and vancomycin are an appropriate empiric antibiotic regimen in an asplenic patient with fever.

A

E

41
Q

Which of the following statements regarding the risk for post-splenectomy sepsis is true?

A. The indication for splenectomy has no bearing on a patient’s risk for developing postsplenectomy sepsis.

B. Adult splenectomy patients have a greater likelihood of developing postsplenectomy sepsis than do children or newborns who require a splenectomy.

C. The risk for postsplenectomy sepsis is highest in the first year after splenectomy, but asplenic patients’ increased risk for developing sepsis persists for approximately 10 years following splenectomy.

D. The risk of sepsis is increased in splenectomy patients due to impaired cellular immunity.

E. None of the above.

A

ANSWER: C

COMMENTS: Asplenic patients (those who have either undergone splenectomy or are functionally asplenic, such as in sickle cell disease) are at an increased risk for a fulminant and potentially rapidly fatal overwhelming bloodstream infection caused by encapsulated organisms.

This is due to impaired bacterial clearance, especially of encapsulated organisms (i.e., S. pneumoniae, Neis- seria meningitidis, and H. influenzae), and impaired humoral (not cellular) immunity.

The risk of developing postsplenectomy sepsis varies based on the indications for splenectomy, the age at which the patient underwent splenectomy (or became functionally asplenic), and the time interval since splenectomy (or functional asplenia).

In terms of indication for splenectomy, patients who have undergone splenectomy for trauma are at the lowest risk; those who have undergone splenectomy for hereditary spherocytosis or immune thrombocytopenic purpura are at intermediate risk; and the greatest risk patients are those who are functionally asplenic due to α-thalassemia, sickle cell disease, or portal hypertension.

Children (especially those under the age of 5 years) are at a greater risk for developing sepsis than are adults, but this may also be confounded by the indications for splenectomy or functional asplenia in this age group.

Overall, the risk for postsplenectomy sepsis is greatest in the first year following splenectomy, but patients who have undergone splenectomy are at an increased risk for sepsis for approximately 10 years following surgery.

Since the year 2000, when universal vaccination with the heptavalent pneumococcal conjugate vaccine (PCV7) in children began, the rates of invasive pneumococcal infection have dropped significantly, and the rates have continued to fall with the development of the 13-valent vaccine (PCV13) in 2010.

Patients who undergo splenectomy or are functionally asplenic should receive vaccinations for the encapsulated organisms prior to splenectomy if it is elective or prior to discharge if performed urgently.

Treatment of postsplenectomy sepsis should be rapid. A fever in any postsplenectomy patient should prompt initiation of antimicrobial therapy as it may be the only sign of an impending fulminant infection.

Postsplenectomy patients, even those who have undergone vaccination for S. pneumoniae, N. meningitidis, and H. influenzae, should be counseled to seek medical attention with any febrile episode.

The most common causative organism is S. pneumoniae, so ceftriaxone, which is also active against H. influenzae and N. meningitidis, is appropriate.

Vancomycin should be administered as well due to concerns for penicillin-resistant pneumococcus and β-lactamase–producing H. influenzae.

Antibiotics should be administered immediately and should not be delayed to obtain cultures.

42
Q

A patient with a recurrent duodenal ulcer is referred for surgical consultation. He has been having abdominal pain for the last 2 years. Fifteen months ago, upper endoscopy showed a duodenal ulcer. The patient was treated with ranitidine, and his condition improved, but the symptoms recurred. Upper endoscopy confirmed a recurrent ulcer, and the result of a Campylobacter-like organism (CLO) test was positive. The patient was treated with a combination of two antibiotics and a PPI for 2 weeks. Which of the following tests best assesses eradication of Helicobacter pylori after completion of treatment?

A. Urea breath test

B. CLO test

C. Biopsy and culture

D. Serum antibody [by enzyme-linked immunosorbent assay (ELISA)]

E. Stool antibody test

A

ANSWER: A

COMMENTS: Surgery for the treatment of peptic ulcers is indicated only in the following circumstances: intractable hemorrhage, perforation, and obstruction.

The patient does not have any of these conditions.

Furthermore, H. pylori infection, the most important pathophysiologic factor in the development of duodenal ulcer, was never adequately treated.

Treatment options for H. pylori infection are numerous, but they must always include an H2 blocker or a PPI plus at least two antibiotics.

The antibiotics most commonly used are amoxicillin, clarithromycin, and metronidazole.

Bismuth-containing regimens have also been used.

Depending on the combination used, the length of treatment varies from 2 to 4 weeks.

Methods of detecting H. pylori can be divided into two categories: invasive and noninvasive.

Biopsy and the CLO test require endoscopy, but all the other tests do not. Like the CLO test, the urea breath test takes advantage of the ability of H. pylori to split urea. However, the urea breath test only requires the patient to “blow,” whereas the CLO test is conducted on a piece of tissue. The serologic test for H. pylori antibody is useful but of limited value in determining the success of the therapy.

There is no stool “antibody” test for H. pylori, but a stool antigen test is available and is as sensitive as the urea breath test.

Since there is no need for repeated endoscopy in this patient, the clinician must consider the relative merits of the noninvasive methods. Because antibody test results may remain positive after treatment, the best choice is the urea breath test, which helps deter- mine the presence of live H. pylori.

43
Q

Which of the following statements regarding anaerobic bacterial infections is true?

A. Anaerobic bacteria are normal inhabitants of the skin and mucous membranes.

B. Bacteroides spp. are the most common isolates in intraabdominal anaerobic infections.

C. If appropriate cultures are obtained, anaerobes are found in more than 75% of intraabdominal abscesses.

D. Proper treatment of anaerobic infections consists of surgical drainage, debridement of necrotic tissue, and appropriate antibiotic therapy.

E. All of the above.

A

ANSWER: E

COMMENTS: Anaerobic bacteria are normal inhabitants of the skin, mucous membranes, and gastrointestinal tract. In fact, anaerobic bacteria outnumber aerobic organisms by more than 10:1 in the oral cavity and by more than 1000:1 in the colon.

Therefore it is not surprising that anaerobes are cultured from up to 90% of intraabdominal abscesses.

The most common pathogens in this group are Bacteroides spp.

Bacteroides fragilis is an important copathogen in the pathogenesis of intraabdominal abscesses.

As with most serious infections, proper treatment involves appropriate drainage of abscesses and debridement of devitalized tissue when present, as well as appropriate antibiotic therapy.

Antibiotics with excellent broad-spectrum anaerobic activity include the carbapenems (imipenem, meropenem, and ertapenem), â-lactam/â-lactamase combinations (ampicillin/sulbactam, ticarcillin/ clavulanate, and piperacillin/tazobactam), and metronidazole.

Although the second-generation cephalosporins (i.e., cefoxitin and cefotetan) and clindamycin also provide anaerobic coverage, over the past decade an increase in resistance of Bacteroides organisms to these agents has been observed.

For example, as many as 30% of B. fragilis isolates are resistant to clindamycin.

44
Q

Which of the following clinical situations or laboratory
results requires systemic antifungal therapy?

A. A single positive blood culture result obtained from an indwelling intravascular catheter

B. Candida identified from a drain

C. Oral candidiasis

D. Candida isolated from a drain culture in a patient who recently underwent surgery for colonic perforation

E. Mucocutaneous candidiasis

A

ANSWER: A

COMMENTS: Candidemia is associated with significant morbidity (e.g., endocarditis, septic arthritis, and ophthalmitis) and mortality (approximately 40%).

Management of candidemia, particularly in patients with intravascular devices, remains controversial.

Although the bloodstream in some patients—usually those who are immunocompetent—spontaneously clears after removal of the intravascular device, other patients—particularly those who are immunosuppressed—have disseminated disease and require systemic antifungal therapy.

There are no accurate diagnostic tests or methods for selecting high-risk patients to determine those who require systemic antifungal therapy.

Therefore all patients with at least one positive blood culture result for Candida should be treated with an antifungal agent.

All nonsurgically implanted lines should be removed, and if continued central venous access is required, a new line should be placed at a new site (not exchanged over a guidewire).

Some would attempt to sterilize the bloodstream without the removal of tunneled catheters or subcutaneous ports. However, in patients with persistent candidemia or septic shock, these devices should also be removed.

Amphotericin B and fluconazole appear to have similar efficacy in the treatment of candidemia. Voriconazole and caspofungin are new antifungal agents that are also effective against Candida. These agents may be particularly useful for non-albicans species such as Candida krusei or Candida glabrata, which are less susceptible to fluconazole.

All patients with candidemia should be evaluated for manifestations of disseminated disease, such as ocular involvement or OM. Candida identified from a surgical drain most likely represents colonization and does not require systemic antifungal therapy.

Mucocutaneous candidiasis can be treated with local nystatin or clotrimazole.

45
Q

A 10-year-old boy who recently emigrated from Mexico has had a 2-day illness characterized by fever, odynophagia, dysphagia, and drooling at the mouth. Physical examination reveals the child to be in a toxic condition with a temperature of 102°F (38.9°C), tachycardia, and tachypnea. There is mild tenderness in the submandibular area and few palpable lymph nodes. The suspected diagnosis is epiglottitis, which is confirmed with a CT scan of the neck. Blood culture results are positive. What kind of organism will probably be seen on Gram stain?

A. Gram-positive cocci in pairs and chains

B. Gram-positive cocci in clusters

C. Slender GNRs

D. Gram-negative coccobacilli

E. Spirochetes

A

ANSWER: D

COMMENTS: The patient has acute epiglottitis, most likely attributable to H. influenzae type B, which is recovered from the blood in up to 100% of cases.

Classically, the patient is a 2- to 4-year-old boy with a short history of fever, irritability, dysphonia, and dysphagia, which can occur at any time of the year.

The widespread use of H. influenzae type B vaccine in developed countries has led to a marked decline in invasive disease with this organism.

However, the disease is still common in developing countries.

Haemophilus species are gram-negative coccobacilli.

Treatment includes early intubation, with plans for a cricothyroidotomy or a tracheotomy if intubation fails, and an antibiotic such as ceftriaxone or ampicillin/sulbactam.

46
Q

Which of the following previously healthy patients scheduled for an operation should undergo human immunodeficiency virus (HIV) antibody testing?

A. A 35-year-old man seen for removal of a lipoma in the anterior triangle of the neck. A routine preoperative complete blood count reveals a WBC count of 4500 cells/ mL with a normal differential, hemoglobin level of 13 g/ dL, and platelet count of 81,000/mL

B. A 40-year-old man seen for repair of an inguinal hernia. Physical examination reveals white, adherent, nonremov- able plaques on the lateral aspect of his tongue

C. A 28-year-old woman seen for the removal of a breast lump in whom a painful vesicular rash along the T8-10 dermatomes develops on the right side

D. A 20-year-old man undergoing nephrectomy for living related-donor transplantation
E. All of the above

A

ANSWER: E

COMMENTS: Several risk groups have been identified in whom HIV testing is indicated, including persons with sexually transmitted diseases and persons in high-risk categories, such as injected drug users, homosexual and bisexual men, hemophiliacs, patients with active tuberculosis (TB), and pregnant women.

Donors of blood or organs should be tested. Certain clinical or laboratory findings should also prompt HIV testing.

Such findings include idiopathic thrombocytopenia, oral hairy leukoplakia, reactivation varicella-zoster virus infection involving more than one dermatome, unexplained oral candidiasis, persistent vulvovaginal candidiasis, and herpes simplex virus infection resistant to treatment.

47
Q

Which of the following is true regarding the management of
hepatic echinococcal disease?

A. Preoperative endoscopic retrograde cholangiopancreatography can be both diagnostic and therapeutic for cyst–biliary communication.

B. Percutaneous drainage of hydatid cysts is contraindicated due to the risk for anaphylaxis.

C. Surgical excision of hydatid cysts need not be preceded by antiparasitic chemotherapy.

D. CT scan is the most sensitive imaging modality to identify communication between hydatid cysts and the biliary tree.

E. Praziquantel is the preferred antiparasitic agent in hydatid cyst disease.

A

ANSWER: A

COMMENTS: Hydatid cyst disease is caused by Echinococcus, with E. multilocularis and E. granulosus being the most common species causing infection in humans.

The definitive hosts of these parasites are dogs, and the intermediate hosts are most commonly sheep or goats; humans are incidental hosts.

Albendazole is the preferred antiparasitic therapy for Echino- coccus.

Mebendazole is also an option, although it has less in vitro activity than albendazole.

Benzimidazole agents interfere with parasite glucose absorption.
The goals of drainage procedures, either surgical or percutane- ous, for hepatic echinococcal cysts are inactivation of the parasites and evacuation and obliteration of the cyst cavity; surgical proce- dures also include the removal of the germinal layer of the cyst cavity.

Radical surgical procedures, such as pericystectomy, partial hepatectomy, or lobectomy, are preferred over less invasive surgical techniques.

PAIR (puncture, aspiration of the cyst, injection of a scolicidal solution such as hypertonic saline, and reaspiration of cyst contents) is generally the preferred percutaneous drainage procedure.

Any drainage procedure should be preceded by the use of a benzimidazole (i.e., albendazole or mebendazole) to sterilize the cyst contents and reduce the risk of anaphylaxis and echinococcal dissemination.

Cystobiliary communication is likely relatively common in hepatic echinococcal disease, but the majority of cases are not clinically apparent.

Magnetic resonance cholangiopancreatography has a sensitivity of 92% in diagnosing intrabiliary rupture, whereas CT scan has only 75% sensitivity. Endoscopic retrograde cholangiopancreatography (ERCP) is also very sensitive (86%–100%) and can be used for therapeutic intervention.

ERCP with sphincterotomy effectively treats cholangitis related to cystobiliary com- munication, and some studies have shown that it can decrease the rate of postoperative biliary fistula formation.

48
Q

A diabetic patient has recently been discharged from the hospital after intracranial bleeding. He is readmitted for aspiration pneumonia. His condition deteriorates rapidly, with hypotension and multiorgan dysfunction. Which of the following treatments is contraindicated?

A. Volume resuscitation

B. Antibiotics

C. Activated protein C

D. Intensive insulin therapy for hyperglycemia

E. Low-dose hydrocortisone

A

ANSWER: C

COMMENTS: Severe sepsis is characterized by multiorgan dysfunction with or without shock and is due to a generalized inflammatory and procoagulant response to infection.

Efforts to improve the outcome with anticytokine therapy along with antibiotics and supportive care have until recently not been associated with improved survival.

Recently, a randomized, double-blind, placebo-controlled multicenter trial evaluating recombinant activated protein C has demonstrated a survival benefit in patients with severe sepsis.

However, activated protein C treatment was associated with an increased risk for bleeding and is contraindicated in patients with recent hemorrhagic stroke. Fluid resuscitation and antibiotics are mainstays in the treatment of sepsis.

Intensive insulin therapy that maintains serum glucose levels at 80 to 110 mg/dL reduces morbidity and mortality in critically ill patients.

The mechanism is unknown, but it is possible that correcting hyper- glycemia may improve neutrophil function.

The use of corticosteroids for sepsis remains controversial. High doses of corticosteroids may in fact worsen outcomes by increasing the frequency of secondary infections.

However, low doses of corticosteroids may be beneficial in septic patients who may have “relative” adrenal insufficiency despite elevated levels of circulating cortisol.

Although the issue is controversial, the use of low-dose hydrocortisone is not contraindicated in this patient.

49
Q

Endocarditis prophylaxis is recommended for which of the following patients?

A. A patient with mitral valve prolapse but without murmur who is undergoing lithotripsy for renal calculi

B. A patient with a history of rheumatic fever and normal cardiac valves who is undergoing prostatic biopsy

C. A patient with a prosthetic aortic valve who is undergo- ing pulmonary resection

D. A patient with severe hypertrophic cardiomyopathy who is undergoing endoscopic retrograde cholangiography for biliary obstruction

E. A patient previously treated for streptococcal endocarditis who is undergoing colonoscopy

A

ANSWER: C

COMMENTS: Antibiotic prophylaxis for endocarditis is recommended for patients with certain cardiac conditions who are undergoing any dental procedure that involves the gingival tissues or periapical region of a tooth and for any procedure involving perforation of the oral mucosa.

In addition, patients undergoing procedures on the respiratory tract or those with skin or soft tissue infections should also receive prophylaxis.

The cardiac conditions associated with the highest risk for adverse outcomes from infective endocarditis for which prophylaxis is indicated prior to the previously listed procedures include prosthetic heart valves, history of infective endocarditis, congenital heart disease (CHD) limited to unrepaired cyanotic CHD, repaired CHD with prosthetic material or devices during the first 6 months after the procedure, repaired CHD with residual defects at the site or adjacent to the site of a prosthesis, and cardiac transplantation recipients with cardiac valvulopathy.

Prophylaxis against viridans group streptococci with a penicillin, cephalosporin, or clindamycin is recommended.

Routine prophylaxis in patients undergoing gastrointestinal or genitourinary procedures is no longer recommended.

50
Q

A patient is infected with HIV. His last CD4+ T-lymphocyte count was 50 cells/mm3, and his viral load was 100,000 copies/ mL. He comes to the hospital with a sudden onset of right hemiparesis. He has been afebrile. A CT scan and magnetic resonance imaging (MRI) of the brain show multiple ring- enhancing lesions in the left cerebral hemisphere. The Toxoplasma IgG antibody test result is positive. He has received pyrimethamine and sulfadiazine for 12 days. Neurologically, the patient is stable. Which of the following is the next best step?

A. Repeat MRI of the brain.

B. Continue the same antibiotic therapy for an additional 10 days and reassess.

C. Switch treatment to pyrimethamine with the addition of clindamycin and reassess whether the patient improves clinically in 10 to 14 days.

D. Add corticosteroids to the treatment regimen.

E. Perform a positron emission tomography (PET) or single-photon emission computed tomography (SPECT) scan.

A

ANSWER: A

COMMENTS: Up to 90% of HIV-infected patients with advanced disease (<100 CD4+ cells/mm3), multiple ring-enhancing lesions, and a positive Toxoplasma IgG antibody have cerebral toxoplasmosis.

Empiric treatment with pyrimethamine, sulfadiazine, and folinic acid is recommended.

Most patients with central nervous system (CNS) toxoplasmosis respond rapidly to this therapy, with nearly 90% of patients demonstrating neurologic improvement at 2 weeks.

Radiographic improvement occurs at a slower pace, with approximately 50% improvement on repeated MRI of the brain occurring within 3 weeks of initiating treatment.

For patients who do not improve by 2 weeks, a brain biopsy is indicated.

Although lymphoma is the most likely alternative diagnosis in patients with acquired immunodeficiency syndrome (AIDS) and CNS lesions, up to 25% of brain biopsy specimens reveal toxoplasmosis.

Thallium-201 (SPECT) or PET scans may provide useful information in that a “cold” lesion revealed by SPECT or hypometabolic lesions seen on PET scanning are consistent with infection.

However, false-positive and false-negative results can occur with these functional imaging studies.

The addition of corticosteroids may be useful in the treatment of increased intracranial pressure.

However, this antiinflammatory effect may make interpretation of clinical and radiographic responses difficult.

51
Q

Match each agent in the left-hand column with one or more mechanisms of antimicrobial action in the right-hand column.

A. Carbapenems

B. Aminoglycosides

C. Quinolones

D. Cephalosporins

E. Vancomycin

a. Impairment of bacterial DNA synthesis
b. Inhibition of cell wall synthesis
c. Disruption of ribosomal protein synthesis
d. Disruption of cell wall cation homeostasis
e. Disruption of the cytoplasmic membrane

A

ANSWER: A-a; B-c, d; C-a; D-b; E-b

COMMENTS: All the antimicrobial agents listed above are bactericidal agents (i.e., their associated mechanisms of action result in bacterial death).

Bacteriostatic agents (e.g., tetracyclines, chloramphenicol, erythromycin, clindamycin, and linezolid) act by prevent- ing bacterial growth but do not result in bacterial death.

They work primarily through inhibition of ribosomal protein synthesis.

Both carbapenems and cephalosporins are â-lactam antibiotics and hence have a similar mode of activity.

Enzymes located within the bacterial cytoplasmic membrane are responsible for peptide cross-linkage.

These enzymes are called penicillin-binding proteins (PBPs) and are the site at which â-lactam drugs bind.

Such binding interferes with bacterial cell wall synthesis and eventually results in cell lysis.

Gram- negative bacteria contain a variable number of various PBPs.

Each â-lactam antibiotic has different affinities for the different PBPs.

Vancomycin is a glycopeptide that also inhibits bacterial cell wall synthesis and assembly. Vancomycin complexes to cell wall precursors and prevents elongation and cross-linkage, thereby making the cell susceptible to lysis. This antibacterial activity is limited to gram- positive organisms.

Aminoglycosides bind irreversibly to the 30S bacterial ribosome and interfere with protein synthesis. For this activity to take place, they must penetrate the cell wall, which occurs optimally under aerobic conditions.

Unlike other antibiotics that inhibit protein synthesis, aminoglycosides are bactericidal. This feature is due to their disruptive effect on calcium and magnesium homeostasis within the cell wall.

Quinolones inhibit topoisomerase II (DNA gyrase) and topoisomerase IV, which impairs DNA synthesis in bacteria.

Appreciation of the mechanism of action of antimicrobials may have a bearing on the selection of alternative therapies when bacterial resistance to the drug of choice develops.

52
Q

Which of the following statements regarding diabetic foot infections is false?

A. Acute diabetic foot infections are often caused by gram-positive organisms.

B. Chronic diabetic foot infections are polymicrobial.

C. To diagnose an infection in a patient with a chronic wound, a foul odor and redness must be present.

D. MRSA infections are associated with a worse outcome.

E. Impaired host defenses allow low-virulence colonizers such as coagulase-negative staphylococci and Corynebac- terium spp. to become pathogens.

A

C

53
Q

Which of the following regarding the treatment of diabetic foot infections is true?

A. Acute diabetic foot infections are caused by monomicrobial gram-negative aerobes.

B. The use of antibiotics for an uninfected chronic wound facilitates wound closure and prevents future infection.

C. Sharp debridement of necrotic or unhealthy tissue prolongs wound healing and removes a potential reservoir for bacteria.

D. Avoiding direct pressure on the wound facilitates healing.

E. The administration of granulocyte-stimulating factors (GSFs) results in faster resolution of the infection.

A

ANSWER: D

COMMENTS: Diabetic patients have a higher risk for foot infections because of factors such as vascular insufficiency, decreased sensation, hyperglycemia, and impairment of the immune system, particularly neutrophil dysfunction.

Deep tissue biopsy of the infected foot is the preferred method of culture.

Acute diabetic foot infections are often caused by monomicrobial aerobic gram-positive cocci (S. aureus and â-hemolytic streptococci, especially group B), whereas patients with chronic wounds and those who have recently received antibiotic therapy generally have polymicrobial gram-positive and gram-negative aerobes and anaerobes within their wound, including enterococci, Enterobacter, obligate anaerobes, and P. aeruginosa.

Initial therapy is usually empiric and based on the severity of infection and available microbiology data (culture results or Gram stain).

A majority of mild infections can be treated with orally dosed antimicrobials directed against aerobic gram- positive cocci.

In patients with more severe infections or extensive chronic infections, parenteral broad-spectrum antibiotics with activity against gram-positive cocci (including MRSA) and gram- negative and obligate anaerobic organisms are warranted.

The diagnosis of infection in patients with chronic wounds includes the presence of purulent secretions (pus) and two or more of the following: redness, warmth, swelling or induration, and pain or tenderness.

MRSA infections are associated with worse outcomes, and impaired host defenses allow low-virulence colonizers such as coagulase-negative staphylococci and Corynebacterium spp. to become pathogens.

In addition to antibiotics, early incision and drainage of abscesses with debridement of devitalized tissue, immobilization, and supportive care are important in the total management of a diabetic foot.

In the presence of significant vascular insufficiency, revascularization of the distal end of the lower extremity may improve healing and prevent amputation.

Radioactive studies using technetium-99 (bone scan) or gallium citrate or indium-labeled leukocyte scans have poor specificity and should not be performed routinely.

MRI has become the imaging study of choice for diagnosing OM. Continued use of antimicrobials is not warranted for the entire time that the wound is open or for the management of clinically uninfected ulceration either to enhance wound healing or as prophylaxis against infection.

Local wound care with sharp debridement of necrotic or unhealthy tissue promotes wound healing and removes a potential reservoir of pathogens.

Avoiding direct pressure on the wound and providing off-loading devices facilitate wound healing.

Administration of granulocyte colony-stimulating factors does not accelerate the resolution of infection but may significantly reduce the need for operative procedures.

54
Q

Match each clinical characteristic or agent in the left-hand column with the correct infecting organism or organisms in the right-hand column. More than one answer may apply.

A. Fibrosing mediastinitis

B. Amphotericin

C. Intertrigo

D. Brain abscess

E. Pelvic mass

a. Candida albicans
b. Nocardia asteroides
c. Actinomyces israelii
d. Cryptococcus neoformans
e. Histoplasma capsulatum

A

ANSWER: A-e; B-a, d, e; C-a; D-b; E-c

COMMENTS: Amphotericin B remains an important agent for the treatment of systemic mycotic infections, including candidiasis, mucormycosis, cryptococcosis, histoplasmosis, coccidioidomycosis, sporotrichosis, and aspergillosis.

Amphotericin B is a fungi- cidal agent. Binding of amphotericin B to ergosterol in the fungal cell membrane alters permeability, with leakage of intracellular ions and macromolecules, leading to cell death.

Adverse events such as infusion reactions and nephrotoxicity are common with the conventional (deoxycholate) form of the drug.

New lipid formulations of amphotericin B have been developed and are associated with a reduction in toxicity without sacrificing efficacy.

Newer triazoles (voriconazole and posaconazole) and echinocandins (caspofungin, micafungin, and anidulafungin) are emerging as alternative broad-spectrum antifungal agents.

Histoplasmosis is predominantly a pulmonary infection caused by Histoplasma capsulatum, a dimorphic fungus endemic to the Mississippi and Ohio River valleys and along the Appalachian Mountains.

Histoplasmosis has been associated with massive enlargement of the mediasti- nal lymph nodes secondary to granulomatous inflammation.

During the healing process, fibrotic tissue can cause postobstructive pneumonia or constriction of the esophagus or superior vena cava and result in dysphagia or superior vena cava syndrome (or both).

Actinomycosis is caused by a group of gram-positive higher-order bacteria that are part of the normal flora found in the oral cavity, gastrointestinal tract, and female genital tract.

Typically, infections with Actinomyces spp. often occur after disruption of mucosal surfaces and lead to oral and cervical disease, pneumonia with empyema, and intraabdominal or pelvic abscesses.

Placement of intrauterine devices has been associated with pelvic abscess secondary to this organism. Sinus tract formation is common as these organisms extend, unrestricted, through tissue planes.

High-dose penicillin and surgical drainage are generally required for cure. Nocardia spp., other higher-order bacteria, are found in soil, organic matter, and water.

Human infection occurs after inhalation or skin inoculation. Chronic pneumonia can occur, usually in immunocompromised patients.

Skin lesions and brain abscesses are common with a disseminated infection.

Prolonged treatment with sulfonamides in combination with other antibiotics is required for cure.

C. neoformans causes meningitis and pulmonary disease.

Infection is common in the setting of immunodeficiency, such as organ transplantation and AIDS, but it may also occur in immunocompetent hosts.

C. albicans is a common inhabitant of the mucous membranes and gastrointestinal tract.

Intertrigo is one form of cutaneous candidiasis that occurs in skinfolds where a warm moist environment exists.

Vesiculopustules develop, enlarge, rupture, and cause maceration and fissuring. Obese and diabetic patients are at risk for the development of candidal intertrigo.

Local care, including nystatin powder, is usually effective.

55
Q

Which of the following statements is correct regarding spontaneous bacterial peritonitis (SBP; primary peritonitis) in a cirrhotic patient?

A. Infection is usually polymicrobial.

B. Ascitic fluid culture results are always positive.

C. The most likely pathogenic mechanism is translocation from the gut.

D. Twenty-one days of antibiotic treatment may be adequate.

E. Infection-related mortality has declined to less than 10%.

A

ANSWER: E

COMMENTS: SBP is a monomicrobial infection, with enteric GNRs accounting for 60%–70% of the episodes of SBP.

E. coli is the most frequently recovered pathogen, followed by K. pneumoniae.

Streptococcal species, including pneumococci and enterococci, are also important pathogens.

Ascitic fluid culture results are negative in many cases, but inoculation of blood culture bottles at the bedside yields bacterial growth in approximately 80% of cases.

SBP most likely develops from the combination of prolonged bacteremia secondary to abnormal host defense, intrahepatic shunting, and impaired bactericidal activity of ascetic fluid.

Transmural migration of gut flora and trans-fallopian spread of vaginal bacteria to the peritoneal space may also occur.

Initial antimicrobial treatment should include coverage against aerobic gram-negative organisms.

A third-generation cephalosporin, such as cefotaxime or ceftriaxone, is a reasonable choice.

The duration of antibiotic treatment is unclear; 2 weeks has been suggested, but shorter courses (5 days) may have similar efficacy.

Although the in-hospital mortality rate approaches 40%, infection-related mortality has declined sig- nificantly (10%).

Unfortunately, the probability of recurrence is 70% at 1 year, with 1- and 2-year survival rates being 30% and 20%, respectively.

56
Q

Which of the following patients with cirrhosis benefit from prophylactic antibiotic therapy to decrease the risk for SBP?

A. Patients awaiting liver transplantation

B. Patients hospitalized with acute gastrointestinal bleeding

C. Patients with ascitic fluid protein levels of greater than 1 g/100 mL

D. Patients who have recovered from a previous episode of SBP

E. Patients with ascitic fluid protein levels of less than 1 g/100 mL

A

ANSWER: C

COMMENTS: Randomized trials have demonstrated that secondary prophylaxis with oral norfloxacin, 400 mg/day, or trimethoprim/ sulfa- methoxazole, one double-strength tablet five times per week decreases the risk for recurrent SBP from 68% to 20%.

However, overall mortality in these patients remains unchanged compared with those not receiving secondary prophylaxis.

Another observation is that long- term quinolone use has been associated with the development of infection with quinolone-resistant bacteria.

In approximately 30%–40% of patients with cirrhosis hospitalized for acute gastrointestinal bleeding, infection develops during the hospitalization.

Norfloxacin (400 mg a day for 7 days) decreases the incidence of infective episodes involving gram-negative bacteria.

The risk for SBP increases tenfold in patients with an ascitic fluid protein concentration of less than 1 g/100 mL fluid.

Norfloxacin, 400mg/day, during hospitalization decreases the incidence of SBP in these patients as well.

Patients hospitalized while awaiting liver transplantation are probably at risk for SBP and may therefore benefit from antibiotic prophylaxis.

Active infection is a contraindication to liver transplantation.

57
Q

Which of the following statements regarding secondary peritonitis is false?

A. It usually occurs because of perforation of an intraabdominal viscus.

B. Carbapenems, aminoglycosides, and fourth-generation cephalosporins have equal efficacy in treatment studies.

C. Increased age, cancer, cirrhosis, and systemic illness are factors that increase the mortality rate.

D. Sequestration of bacteria within fibrin clots leads to intraabdominal abscess formation.

E. The most common organism cultured from the abdomen is E. coli.

A

ANSWER: B

COMMENTS: Secondary peritonitis usually occurs because of perforation of an intraabdominal viscus such as perforated peptic ulcer, appendix, or diverticulum or penetrating gastrointestinal trauma.

The infection is polymicrobial, with facultative aerobes and anaerobes acting synergistically.

One study revealed an average of 2.5 anaerobes and 2 facultative aerobes identified per case of secondary peritonitis.

E. coli is the most common isolate in culture. Bacteroides spp. are the most frequent anaerobes cultured from abdominal infections.

About 1012 bacteria reside in the colon per gram of feces, with 90% of these bacteria being anaerobic organisms.

Any process that impairs immunologic function or is associated with general debilitation increases mortality.

Age, cancer, hepatic cirrhosis, and the presence of a systemic illness have been shown to increase mortality.

One of the defense mechanisms of the peritoneal cavity is the production of fibrin to sequester bacteria for limiting systemic spread.

Such sequestration leads to the formation of intraabdominal abscesses, which generally require drainage for cure.

Treatment of secondary peritonitis requires surgical intervention for removal of the source of infection and systemic antibiotics for eradication of residual bacteria.

Antibiotics selected should include agents with broad-spectrum coverage for fac- ultative aerobes, gram-negative bacilli, and anaerobes.

Carbapenems are a good empiric choice for treatment.

Aminoglycosides and fourth- generation cephalosporins lack anaerobic activity.

58
Q

Which of the following statements regarding cytomegalovirus (CMV) infection and solid organ transplantation is false?

A. Symptomatic infection occurs 2 to 6 months after transplantation.

B. Patients being treated for acute rejection are at an increased risk for the development of symptomatic CMV infection.

C. Transmission can occur through the donor organ.

D. Reactivation of latent infection is associated with the greatest risk for the development of severe disease.

E. CMV infection may be associated with premature atherosclerosis in cardiac transplant patients.

A

ANSWER: D

COMMENTS: CMV is the most important pathogen affecting recipients of solid organ transplants.

Symptomatic CMV disease may develop in as many as 50% of allograft recipients, usually 2 to 6 months after transplantation.

CMV-seronegative recipients who are primarily infected are at the greatest risk for the development of severe CMV disease.

Primary infection can occur through the donor organ, unscreened blood products, or intimate contact with a viral shedder. Reactivation of latent infection is less likely to cause severe disease.

Patients receiving muromonab-CD3 (OKT3)/antilymphocyte globulin (ALG) therapy for acute rejection also appear to be at risk for the development of CMV disease.

In addition to clinical disease directly attributable to CMV infection, CMV has indirect immunomodulatory activity.

Symptomatic CMV infections are associated with an increased incidence of bacterial infections and opportunistic infections such as aspergillosis and Pneumocystis carinii pneumonia.

In heart transplant patients, acute rejection and accelerated atherosclerosis are associated with CMV infection.

Ganciclovir is the most commonly used agent for the prevention of CMV infection and disease; however, there is growing concern regarding the emergence of ganciclovir resistance.

59
Q

A 28-year-old man who sustained closed-head trauma in a motor vehicle accident a month earlier comes to the emergency department with a 3-day history of progressive headache, fever, and confusion. His wife reports the recent onset of clear drainage from his left naris. Physical examination reveals a temperature of 102°F (38.9°C), a stiff neck, and no rash. Which of the following statements concerning the patient is true?

A. He most likely has bacterial meningitis secondary to S. aureus.

B. Antiretroviral prophylaxis has been beneficial in preventing bacterial meningitis after head trauma.

C. Empiric antibiotics should include an extended-spectrum cephalosporin and vancomycin.

D. Corticosteroid administration with antibiotics is not indicated.

E. He requires immediate
surgical intervention for repair of cerebrospinal fluid leakage.

A

ANSWER: C
COMMENTS: The patient probably sustained a basilar skull fracture and a dural rent, with subsequent development of a dural fistula from the subarachnoid space and nasal cavity or paranasal sinuses.

Cerebrospinal fluid rhinorrhea may occur and can easily be diagnosed by detecting the presence of â2-transferrin in nasal secretions.

In patients with known basilar skull fracture, cerebrospinal fluid rhinorrhea develops in approximately 10% of the cases.

Of these patients, bacterial meningitis develops in up to 30%. S. pneumoniae is the most common pathogen (65% of cases).

Other organisms, such as H. influenzae, N. meningitidis, and S. aureus, account for the remaining cases.

Empiric treatment should include an extended-spectrum cephalosporin (ceftriaxone, cefotaxime, or cefepime) and vancomycin since the incidence of â-lactam–resistant pneumococci is increasing.

Prophylactic antibiotics have no proven benefit and may predispose to meningitis from antibiotic- resistant gram-negative bacteria.

A recent prospective study demonstrated a survival advantage in patients with pneumococcal meningitis who received corticosteroids before or at the time of antibiotic administration.

Spontaneous closure of the dural fistula is less likely in patients with a delayed manifestation of cerebrospinal fluid leakage with meningitis, and surgical repair is indicated.

Diagnostic studies to identify the site of the fistula and treatment of any CNS infection should be completed before surgical intervention.

60
Q

Which of the following statements regarding hepatitis C virus (HCV) infection is false?

A. The prevalence of HCV infection in HCWs is similar to that in the general population.

B. Chronic HCV infection occurs in 75%–85% of patients after acute infection.

C. Hepatic failure because of chronic HCV infection is the most common indication for liver transplantation.

D. Pegylated interferon plus ribavirin is an effective therapy for most patients with chronic HCV infection.

E. Factors associated with the development of cirrhosis include male gender, alcohol use, and coinfection with HIV.

A

ANSWER: D

COMMENTS: Persons with acute HCV infection are typically asymptomatic (60%–70%) or have a mild clinical illness.

Fulminant hepatitis is rare. Chronic HCV infection develops in approximately 75%–80% of persons with acute HCV infection.

Cirrhosis develops in 10%–20% of chronically infected individuals, usually after more than 20 years of infection.

Liver failure from chronic HCV infection has become the leading indication for liver transplantation.

Increased alcohol use, male gender, HIV coinfection, and HCV genotype 1 are associated with more severe liver disease.

Hepatocellular carcinoma can be a late complication in 1%–2% of patients with cirrhosis.

Antiviral therapy is recommended for individuals at an increased risk for progressive liver disease, as demonstrated by persistently elevated serum transaminase levels, detectable HCV RNA levels, and moderate inflammation in liver biopsy specimens.

The prevalence of HCV infection is highest in injected drug users and patients undergoing hemodialysis.

Overall, nearly 2% of the U.S. population has persistent HCV infection.

Although trans- mission of HCV to HCWs occurs after approximately 3% of needlestick exposures involving HCV-infected patients, the prevalence of HCV infection in HCWs, including surgeons, is like the general population.

61
Q

Which statement about Mycobacterium tuberculosis treatment and prophylaxis is true?

A. Two-drug treatment with INH and rifampin (RIF) for 9 months is standard therapy for active pulmonary TB.

B. Treatment failure can be due to drug resistance or nonadherence.

C. HIV-infected individuals require prolonged therapy for active TB.

D. INH prophylaxis for latent TB is given for at least 12 months.

E. INH prophylaxis should not be given to individuals with recent conversion from purified protein derivative (PPD)-negative to PPD-positive status

A

ANSWER: B

COMMENTS: Recent Centers for Disease Control and Prevention (CDC) guidelines recommend that all patients with active pulmonary TB receive four-drug therapy consisting of INH, RIF, pyrazinamide, and ethambutol for the initial 2 months of treatment. For patients with drug-susceptible TB and negative sputum test results after 2 months of therapy, treatment can be completed with 4 months of INH and RIF.

Extrapulmonary disease requires 6 to 9 months of treatment, except for meningitis, which is treated for 1 year. HIV-infected individuals are treated similar to non–HIV-infected patients with TB.

However, significant drug–drug interactions may occur with antiretroviral agents and TB drugs and may alter therapeutic decisions.

Treatment failures are generally due to nonadherence by patients to multidrug regimens.

Currently, local health departments have directly observed therapy programs to improve compliance with and completion of anti-TB medication regimens.

Another cause of treatment failure is infection with multidrug-resistant strains of M. tuberculosis.

Conditions associated with a higher rate of resistance include TB in those known to have a higher prevalence of drug resistance, such as Asians or Hispanics and previously treated individuals; the persistence of culture-positive sputum after 2 months of therapy; and known exposure to drug-resistant TB.

Certain individuals are at considerable risk for the development of active TB once infected (latent TB).

TB skin testing (Mantoux/ PPD) is useful for identifying latent TB in high-risk individuals.

Three cut points have been recommended for defining a positive tuberculin reaction: greater than 5 mm, greater than 10 mm, and greater than 15 mm of induration.

Persons considered at highest risk (>5 mm of induration) include individuals with HIV infection, recent contacts with TB patients, and organ transplant patients. Individuals also at risk (>10-mm induration) include injectable drug users, residents of nursing homes and prisons, hospital employees, and recent immigrants from countries with a high prevalence of TB.

These individuals, who are at a considerable risk for the development of active TB once infected, should receive 9 months of INH therapy.

62
Q

Which of the following is true regarding the bacteriology of
vascular graft infections?

A. S. epidermidis is the most commonly isolated organism.

B. Fungal graft infections are uncommon, but when they do occur, they are most common in immunocompromised patients.

C. Gram-negative organisms are implicated most frequently in thoracic aortic and carotid artery graft infections.

D. Gram-negative infections are often less virulent and have fewer major complications than do gram-positive infections.

E. None of the above.

A

B

63
Q

Which of the following clinical scenarios raises concern for vascular graft infection?

A. New-onset gastrointestinal bleeding in a patient with a history of abdominal aortic endograft placement

B. The presence of a draining sinus tract 3 cm distal to a forearm arteriovenous graft for hemodialysis

C. A slowly growing pulsatile mass in the groin of a patient with a history of an aortofemoral bypass graft

D. CT scan demonstrating the presence of a fluid collection adjacent to an abdominal aortic graft
E. All of the above

A

ANSWER: E

COMMENTS: Vascular graft infections can manifest in a variety of ways, and surgeons must maintain a high index of suspicion for graft infection.

There are multiple mechanisms by which vascular grafts can become infected: perioperative contamination (including from lymphatic disruption), seeding from bacteremia, contiguous spread from an adjacent infectious process, and erosion of the graft into the gastrointestinal or genitourinary tracts.

Grafts are more prone to infection than natural tissue since bacteria adhere to graft material and form a biofilm that resists the body’s natural immunologic defenses.

The most likely organisms in vascular graft infections are gram-positive cocci such as S. aureus (the most common pathogen) and S. epidermidis, but the incidence of GNR infections is significantly increased in abdominal aortic, aortofemoral, and infrainguinal vascular graft infections.

Gram-negative infections are more virulent than gram-positive infections, because gram-negative organisms produce proteases, elastases, and other destructive enzymes, which can cause anastomotic disruption and vessel rupture.

In the vast majority of cases of vascular graft infection, the prosthetic material must be excised for complete eradication of the infection.

64
Q

Suspicion of OM in a diabetic foot ulcer should be raised in
all of the following except:

A. A deep ulcer that overlies a bony prominence

B. An ulcer that does not heal after 2 weeks of appropriate therapy

C. A patient with a swollen foot and a history of foot ulceration

D. Unexplained high WBC count or inflammatory markers in a patient with a diabetic foot ulcer

E. Evidence of cortical erosion and periosteal reaction on plain radiography

A

B

65
Q

Which of the following is true regarding OM in a diabetic foot?

A. A nuclear medicine-tagged WBC scan is the best way to diagnose OM.

B. The only reported successful treatment of OM includes resection of the infected bone.

C. A presumptive diagnosis of OM cannot be made even if bone destruction is seen on plain film underneath an ulcer.

D. A bone biopsy is often difficult to perform and invasive and should be avoided.

E. Selected patients may benefit from implanted antibiotics, hyperbaric oxygen therapy, or revascularization.

A

ANSWER: E

COMMENTS: OM impairs healing of the wound and acts as a nidus for recurrent infection.

It should be suspected in any deep or extensive ulcer, in one that overlies a bony prominence, and in an ulcer that does not heal after 6 weeks of appropriate therapy.

In addition, concern for OM is raised in a patient with a swollen foot and a history of foot ulceration, the presence of a “sausage toe” (red, swollen digit), unexplained high WBC count, or inflammatory markers.

Bone destruction underneath an ulcer seen on radiographs or probing of an ulcer down to bone is OM until proved otherwise.

MRI is the most useful available imaging modality to diagnose OM, as well as to characterize any underlying soft tissue infection.

The “gold standard” for diagnosis of OM remains isolation of bacteria from a bone sample with concomitant histologic findings of inflammatory cells and osteonecrosis.

When treating a diabetic foot infection, if there are no hard signs to indicate the presence of OM and plain radiographs do not demonstrate any evidence of bone pathology, the patient should be treated for about 2 weeks for the soft tissue infection.

If there is a persistent concern for OM, plain films should be repeated in 2 to 4 weeks to look for evidence of cortical erosion, periosteal reaction, or mixed radiolucency and sclerosis.

Radioisotope scans are more sensitive than plain radiographs for diagnosis but are expensive and can be time consuming.

If findings on plain films are only consistent with but not characteristic of OM, the clinician should consider the following:

(1) additional imaging studies—MRI is preferred but nuclear medicine scans with leukocyte or immunoglobulin techniques would be the second choice;
(2) empiric treatment for an additional 2 to 4 weeks with repeated radiographs to look for progression of bone changes; and
(3) bone biopsy (operative or percutaneous fluoroscopic or CT guidance), especially if the etiologic pathogen or susceptibilities need to be established.

Some physicians would perform biopsies for midfoot or hindfoot lesions because these are more difficult to treat and lead to higher-level amputations.

Traditionally, resection of a bone with chronic OM was necessary for cure; however, some nonrandomized case series report clinical success in 65%–80% of patients treated nonoperatively with prolonged (3 to 6 months) antibiotic therapy.

When treatment of OM fails, the clinician should consider whether the original diagnosis was correct; whether there is any remaining necrotic or infected bone or surgical hardware that needs to be removed; and whether the antimicrobials selected were appropriate, achieved an effective concentration within the bone, and were used for a sufficient duration.

Selected patients may benefit from implanted antibiotics, hyperbaric oxygen therapy, revascularization, long-term or intermittent antibiotic administration, or amputation.

66
Q

Which of the following regarding hospital-acquired pneumo- nia (HAP), VAP, and health care–associated pneumonia (HCAP) is false?

A. They are the most common nosocomial infections.

B. They are usually caused by aerobic gram-negative bacilli.

C. They are rarely due to viral or fungal pathogens in immunocompetent patients.

D. Infection resulting from aspiration is usually due to anaerobes.

E. Gram-positive coccal isolates are more common patients with head trauma.

A

A

67
Q

Which of the following are risk factors for HAP, VAP, or
HCAP caused by multidrug-resistant pathogens?

A. Hospitalization for 5 or more days

B. Antimicrobial therapy or hospitalization in the preceding 90 days

C. Home wound care

D. Immunosuppressive disease or therapy

E. All of the above

A

ANSWER: E

COMMENTS: HAP, HCAP, and VAP are the second most common nosocomial infections after UTI. They result in significant morbidity and mortality.

They are due to a wide spectrum of bacterial pathogens and are often polymicrobial, especially in patients with acute respiratory distress syndrome.

They are rarely due to viral or fungal agents in immunocompetent patients.

Isolation of Candida from endotracheal aspirates of immunocompetent patients usually represents colonization.

Common pathogens include aerobic gram-negative bacilli, including P. aeruginosa, E. coli, K. pneumoniae, and Acineto- bacter spp.

There has been an emergence of pneumonia associated with gram-positive cocci (S. aureus, particularly MRSA), and it is more commonly seen in diabetics, patients with head trauma, and those hospitalized in the ICU.

Infection with anaero- bic organisms may follow aspiration in nonintubated patients but is rare in VAP.
Early-onset HAP or VAP occurring within the first 4 days of hospitalization carries a better prognosis than do late-onset infections (5 days or more), which are more likely to be due to multidrug- resistant bacterial pathogens and result in increased morbidity and mortality.

Additional risk factors for multidrug-resistant bacterial pathogens such as Pseudomonas, Acinetobacter spp., MRSA, and K. pneumoniae include antimicrobial therapy or hospitalization in the preceding 90 days, a high frequency of antibiotic resistance in the community or in the specific hospital unit, and immunosuppressive disease or therapy.

Risk factors for multidrug-resistant pathogens in patients with HCAP include residence in a nursing home or long- term care facility, home infusion therapy, chronic dialysis within 30 days, home wound care, and a family member with a multidrug-resistant pathogen.

P. aeruginosa is the most common gram-negative bacterial pathogen that causes multidrug-resistant HAP/VAP, with some iso- lates being susceptible only to polymyxin B. Most MRSA infections are treated successfully with linezolid, although MRSA isolates resistant to linezolid are emerging.

Early administration of a broad-spectrum antibiotic in adequate doses and deescalation of the initial antibiotic therapy based on cultures and clinical response are essential.

Failure to adequately treat the infection because of delayed initiation of appropriate therapy has been associated with increased mortality.

Guidelines have been established by the American Thoracic Society and the Infectious Disease Society of America for empiric therapy in immunocompetent adults with bacterial causes of HAP, VAP, or HCAP; treatment should include either ceftriaxone, a fluoroquinolone, ampicillin/sulbactam, or ertapenem if there is no suspicion of a multidrug-resistant pathogen.

68
Q

Which of the following antimicrobial agents is considered safe in pregnancy?

A. Ganciclovir

B. Albendazole

C. Ketoconazole

D. Streptomycin

E. Erythromycin

A

ANSWER: E

COMMENTS: Multiple antimicrobial agents are contraindicated in pregnancy.

Of the answer choices listed above, erythromycin is the only agent considered safe.

Most penicillins and cephalosporins are safe as well.

Ganciclovir, ketoconazole, and albendazole are teratogenic in the first trimester.

Streptomycin has been shown to cause ototoxicity in fetuses.

69
Q

He developed techniques of sterilization critical to oenology and identified several bacteria responsible for human illnesses, including Staphylococcus, Streptoccocus, and pneumococcus.

A. Ignaz Semmelweis

B. Robert Koch

C. Louis Pasteur

D. Joseph Lister

A

C. Louis Pasteur

70
Q

True statement/s in source control include/s:

A. The first operation to use this principle was cholecystectomy.

B. Pioneered by Ignaz Semmelweis

C. The first operation to use this principle was herniorrhaphy.

D. Pioneered Charles McBurney

A

D. Pioneered Charles McBurney

71
Q

Presence of microorganisms in the bloodstream and an inflammatory response to that presence is termed:

A. Sepsis
B. Severe Sepsis
C. Infection
D. SIRS

A

C. Infection

72
Q

Sepsis with need for ventilatory support or oliguria unresponsive to aggressive fluid resuscitation should be considered to have developed

A. Bacteremia

B. Infection

C. Severe sepsis

D. Septics shock

A

C. Severe sepsis

73
Q

Gram-negative aerobic bacilli that commonly cause infection in the surgical patient includes:

A. Bacteroides fragilis

B. Enteroccocus faecium

C. E. faecalis

D. Eschericia coli

A

D. Eschericia coli

74
Q

Common gram-positive anaerobes in surgical patients include/s:

A. Clostridium perfringes

B. Streptoccocus pyogenes

C. Eschericia coli

D. Serratia marcescens

A

A. Clostridium perfringes

75
Q

Main treatment for abscess include/s which of the following:

A. Drainage

B. Antibiotics

C. Warm compress

D. Analgesics

A

A. Drainage

76
Q

True statements in aggressive soft tissue infections

A. The extremities, perineum, and torso are most commonly affected

B. Pain at the site of infection out of proportion to the physical manifestations

C. Presence of skin changes (bronze hue or brawny induration), blebs, or crepitus

D. All of the above.

A

D. All of the above.

77
Q

Effective treatment for incisional surgical site infection include/s:

A. Topical antibiotics and antiseptics to hasten wound healing.

B. Broad-spectrum antibiotics is required in all cases

C. Incision and drainage

D. All of the above

A

C. Incision and drainage

78
Q

Microbial peritonitis that occurs subsequent to contamination of the peritoneal cavity due to perforation or severe inflammation and infection of an intraabdominal organ

A. Persistent

B. Tertiary

C. Secondary

D. Primary

A

C. Secondary

79
Q

Primary microbial peritonitis treatment include/s

A. 14-21 days of antibiotics

B. Removal of infected organ/s

C. Ultrasound-guided paracentesis

D. Laparotomy

E. All of the above

A

A. 14-21 days of antibiotics

80
Q

Soft tissue infections that are effectively treated with antibiotics alone include/s

A. Furuncle

B. Boils

C. Cellulitis

D. Carbuncle

A

C. Cellulitis